Как найти логическое значение высказывания

Алгебра логики

Алгебра логики

Алгебра логики (англ. algebra of logic) — один из основных разделов математической логики, в котором методы алгебры используются в логических преобразованиях.

Основоположником алгебры логики является английский математик и логик Дж. Буль (1815–1864), положивший в основу своего логического учения аналогию между алгеброй и логикой. Любое высказывание он записывал с помощью символов разработанного им языка и получал «уравнения», истинность или ложность которых можно было доказать, исходя из определенных логических законов, таких как законы коммутативности, дистрибутивности, ассоциативности и др.

Современная алгебра логики является разделом математической логики и изучает логические операции над высказываниями с точки зрения их истинностного значения (истина, ложь). Высказывания могут быть истинными, ложными или содержать истину и ложь в разных соотношениях.

Логическое высказывание — это любое повествовательное предложение, в отношении которого можно однозначно утверждать, что его содержание истинно или ложно.

Например, «3 умножить на 3 равно 9», «Архангельск севернее Вологды» — истинные высказывания, а «Пять меньше трех», «Марс — звезда» — ложные.

Очевидно, что не всякое предложение может быть логическим высказыванием, т. к. не всегда есть смысл говорить о его ложности или истинности. Например, высказывание «Информатика — интересный предмет» неопределенно и требует дополнительных сведений, а высказывание «Для ученика 10-А класса Иванова А. А. информатика — интересный предмет» в зависимости от интересов Иванова А. А. может принимать значение «истина» или «ложь».

Кроме двузначной алгебры высказываний, в которой принимаются только два значения — «истинно» и «ложно», существует многозначная алгебра высказываний. В такой алгебре, кроме значений «истинно» и «ложно», употребляются такие истинностные значения, как «вероятно», «возможно», «невозможно» и т. д.

В алгебре логики различаются простые (элементарные) высказывания, обозначаемые латинскими буквами (A, B, C, D, …), и сложные (составные), составленные из нескольких простых с помощью логических связок, например таких, как «не», «и», «или», «тогда и только тогда», «если … то». Истинность или ложность получаемых таким образом сложных высказываний определяется значением простых высказываний.

Обозначим как А высказывание «Алгебра логики успешно применяется в теории электрических схем», а через В — «Алгебра логики применяется при синтезе релейно-контактных схем».

Тогда составное высказывание «Алгебра логики успешно применяется в теории электрических цепей и при синтезе релейно-контактных схем» можно кратко записать как А и В; здесь «и» — логическая связка. Очевидно, что поскольку элементарные высказывания А и В истинны, то истинно и составное высказывание А и В.

Каждая логическая связка рассматривается как операция над логическими высказываниями и имеет свое название и обозначение.

Логических значений всего два: истина (TRUE) и ложь (FALSE). Это соответствует цифровому представлению — 1 и 0. Результаты каждой логической операции можно записать в виде таблицы. Такие таблицы называют таблицами истинности.

Основные операции алгебры логики

1. Логическое отрицание, инверсия (лат. inversion — переворачивание) — логическая операция, в результате которой из данного высказывания (например, А) получается новое высказывание (не А), которое называется отрицанием исходного высказывания, обозначается символически чертой сверху ($A↖{-}$) или такими условными обозначениями, как ¬, ‘not’, и читается: «не А», «А ложно», «неверно, что А», «отрицание А». Например, «Марс — планета Солнечной системы» (высказывание А); «Марс — не планета Солнечной системы» ($A↖{-}$); высказывание «10 — простое число» (высказывание В) ложно; высказывание «10 — не простое число» (высказывание B ) истинно.

Операция, используемая относительно одной величины, называется унарной. Таблица значений данной операции имеет вид

A ¬A
истина ложь
ложь истина

или

Высказывание $A↖{-}$ ложно, когда А истинно, и истинно, когда А ложно.

Геометрически отрицание можно представить следующим образом: если А — это некоторое множество точек, то $A↖{-}$ — это дополнение множества А, т. е. все точки, которые не принадлежат множеству А.

2. Конъюнкция (лат. conjunctio — соединение) — логическое умножение, операция, требующая как минимум двух логических величин (операндов) и соединяющая два или более высказываний при помощи связки «и» (например, «А и В»), которая символически обозначается с помощью знака ∧ (А ∧ В) и читается: «А и В». Для обозначения конъюнкции применяются также следующие знаки: А ∙ В; А & В, А and В, а иногда между высказываниями не ставится никакого знака: АВ. Пример логического умножения: «Этот треугольник равнобедренный и прямоугольный». Данное высказывание может быть истинным только в том случае, если выполняются оба условия, в противном случае высказывание ложно.

Таблица истинности операции имеет вид

A B A ∧ B
истина ложь ложь
ложь истина ложь
ложь ложь ложь
истина истина истина

или

A B A ∧ B
1 0 0
0 1 0
0 0 0
1 1 1

Высказывание АВ истинно только тогда, когда оба высказывания — А и В истинны.

Геометрически конъюнкцию можно представить следующим образом: если А, В — это некоторые множества точек, то АВ есть пересечение множеств А и В.

3. Дизъюнкция (лат. disjunction — разделение) — логическое сложение, операция, соединяющая два или более высказываний при помощи связки «или» (например, «А или В»), которая символически обозначается с помощью знака ∨ В) и читается: «А или В». Для обозначения дизъюнкции применяются также следующие знаки: А + В; А or В; А | B. Пример логического сложения: «Число x делится на 3 или на 5». Это высказывание будет истинным, если выполняются оба условия или хотя бы одно из условий.

Таблица истинности операции имеет вид

A B AB
истина ложь истина
ложь истина истина
ложь ложь ложь
истина истина истина

или

A B AB
1 0 1
0 1 1
0 0 0
1 1 1

Высказывание А В ложно только тогда, когда оба высказывания — А и В ложны.

Геометрически логическое сложение можно представить следующим образом: если А, В — это некоторые множества точек, то АВ — это объединение множеств А и В, т. е. фигура, объединяющая и квадрат, и круг.

4. Дизъюнкция строго-разделительная, сложение по модулю два — логическая операция, соединяющая два высказывания при помощи связки «или», употребленной в исключающем смысле, которая символически обозначается с помощью знаков ∨ ∨ или ⊕ (А ∨ ∨ В, АВ) и читается: «либо А, либо В». Пример сложения по модулю два — высказывание «Этот треугольник тупоугольный или остроугольный». Высказывание истинно, если выполняется какое-то одно из условий.

Таблица истинности операции имеет вид

А В А B
истина ложь истина
ложь истина истина
ложь ложь ложь
истина истина ложь

или

А В А B
1 0 1
0 1 1
0 0 0
1 1 0

Высказывание А ⊕ В истинно только тогда, когда высказывания А и В имеют различные значения.

5. Импликация (лат. implisito — тесно связываю) — логическая операция, соединяющая два высказывания при помощи связки «если…, то» в сложное высказывание, которое символически обозначается с помощью знака → (АВ) и читается: «если А, то В», «А влечет В», «из А следует В», «А имплицирует В». Для обозначения импликации применяется также знак ⊃ (A ⊃ B). Пример импликации: «Если полученный четырехугольник квадрат, то около него можно описать окружность». Эта операция связывает два простых логических выражения, из которых первое является условием, а второе — следствием. Результат операции ложен только тогда, когда предпосылка есть истина, а следствие — ложь. Например, «Если 3 * 3 = 9 (А), то Солнце — планета (В)», результат импликации А → В — ложь.

Таблица истинности операции имеет вид

А В А В
истина ложь ложь
ложь истина истина
ложь ложь истина
истина истина истина

или

А В А В
1 0 0
0 1 1
0 0 1
1 1 1

Для операции импликации справедливо утверждение, что из лжи может следовать все что угодно, а из истины — только истина.

6. Эквивалентность, двойная импликация, равнозначность (лат. aequalis — равный и valentis — имеющий силу) — логическая операция, позволяющая из двух высказываний А и В получить новое высказывание А ≡ В, которое читается: «А эквивалентно B». Для обозначения эквивалентности применяются также следующие знаки: ⇔, ∼. Эта операция может быть выражена связками «тогда и только тогда», «необходимо и достаточно», «равносильно». Примером эквивалентности является высказывание: «Треугольник будет прямоугольным тогда и только тогда, когда один из углов равен 90 градусам».

Таблица истинности операции эквивалентности имеет вид

А В А В
истина ложь ложь
ложь истина ложь
ложь ложь истина
истина истина истина

или

А В А В
1 0 0
0 1 0
0 0 1
1 1 1

Операция эквивалентности противоположна сложению по модулю два и имеет результат «истина» тогда и только тогда, когда значения переменных совпадают.

Зная значения простых высказываний, можно на основании таблиц истинности определить значения сложных высказываний. При этом важно знать, что для представления любой функции алгебры логики достаточно трех операций: конъюнкции, дизъюнкции и отрицания.

Сложение по модулю два А ⊕ В $(A↖{-} ∧B) ∧ (A ∧ B↖{-})$
Импликация А → В $A↖{-} ∨ B$
Эквивалентность А ∼ В $(A↖{-} ∧ B↖{-}) ∨ (A ∧ B)$

Приоритет выполнения логических операций следующий: отрицание («не») имеет самый высокий приоритет, затем выполняется конъюнкция («и»), после конъюнкции — дизъюнкция («или»).

С помощью логических переменных и логических операций любое логическое высказывание можно формализовать, т. е. заменить логической формулой. При этом элементарные высказывания, образующие составное высказывание, могут быть абсолютно не связаны по смыслу, но это не мешает определять истинность или ложность составного высказывания. Например, высказывание «Если пять больше двух (А), то вторник всегда наступает после понедельника (В)» — импликация А В, и результат операции в данном случае — «истина». В логических операциях смысл высказываний не учитывается, рассматривается только их истинность или ложность.

Рассмотрим, например, построение составного высказывания из высказываний А и В, которое было бы ложно тогда и только тогда, когда оба высказывания истинны. В таблице истинности для операции сложения по модулю два находим: 1 ⊕ 1 = 0. А высказывание может быть, например, таким: «Этот мяч полностью красный или полностью синий». Следовательно, если утверждение А «Этот мяч полностью красный» — истина, и утверждение В «Этот мяч полностью синий» — истина, то составное утверждение — ложь, т. к. одновременно и красным, и синим мяч быть не может.

Примеры решения задач

Пример 1. Определить для указанных значений X значение логического высказывания ((X > 3) ∨ (X < 3)) → (X < 4) :

1) X = 1; 2) X = 12; 3) X = 3.

Решение. Последовательность выполнения операций следующая: сначала выполняются операции сравнения в скобках, затем дизъюнкция, и последней выполняется операция импликации. Операция дизъюнкции ∨ ложна тогда и только тогда, когда оба операнда ложны. Таблица истинности для импликации имеет вид

A B A → B
1 0 0
0 1 1
0 0 1
1 1 1

Отсюда получаем:

1) для X = 1:

((1 > 3) ∨ (1 < 3)) → (1 < 4) = ложь ∨ истина → истина = истина → истина = истина;

2) для X = 12:

((12 > 3) ∨ (12 < 3) → (12 < 4) = истина ∨ ложь → ложь = истина → ложь = ложь;

3) для X = 3:

((3 > 3) ∨ (3 < 3)) → (3<4) = ложь ∨ ложь → истина = ложь → истина = истина.

Пример 2. Указать множество целых значений X, для которых истинно выражение ¬((X > 2) → (X > 5)) .

Решение. Операция отрицания применена ко всему выражению ((X > 2) → (X > 5)) , следовательно, когда выражение ¬((X > 2) → (X > 5)) истинно, выражение ((X > 2) →(X > 5)) ложно. Поэтому необходимо определить, для каких значений X выражение ((X > 2) → (X > 5)) ложно. Операция импликации принимает значение «ложь» только в одном случае: когда из истины следует ложь. А это выполняется только для X = 3; X = 4; X = 5.

Пример 3. Для каких из приведенных слов ложно высказывание ¬(первая буква гласная ∧ третья буква гласная) ⇔ строка из 4 символов? 1) асса; 2) куку; 3) кукуруза; 4) ошибка; 5) силач.

Решение. Рассмотрим последовательно все предложенные слова:

1) для слова асса получим: ¬(1 ∧ 0) ⇔ 1, 1 ⇔ 1 — высказывание истинно;

2) для слова куку получим: ¬ (0 ∧ 0) ⇔ 1, 1 ⇔ 1 — высказывание истинно;

3) для слова кукуруза получим: ¬ (0 ∧ 0) ⇔ 0, 1 ⇔ 0 — высказывание ложно;

4) для слова ошибка получим: ¬ (1 ∧ 1) ⇔ 0, 0 ⇔ 0 — высказывание истинно;

5) для слова силач получим: ¬ (0 ∧ 0) ⇔ 1, 1 ⇔ 0 — высказывание ложно.

Логические выражения и их преобразование

Под логическим выражением следует понимать такую запись, которая может принимать логическое значение «истина» или «ложь». При таком определении среди логических выражений необходимо различать:

  • выражения, которые используют операции сравнения («больше», «меньше», «равно», «не равно» и т. п.) и принимают логические значения (например, выражение а > b , где а = 5 и b = 7, равно значению «ложь»);
  • непосредственные логические выражения, связанные с логическими величинами и логическими операциями (например, A ∨ В ∧ С, где А = истина, B = ложь и C = истина).

Логические выражения могут включать в себя функции, алгебраические операции, операции сравнения и логические операции. В этом случае приоритет выполнения действий следующий:

  1. вычисление существующих функциональных зависимостей;
  2. выполнение алгебраических операций (вначале умножение и деление, затем вычитание и сложение);
  3. выполнение операций сравнения (в произвольном порядке);
  4. выполнение логических операций (вначале операции отрицания, затем операции логического умножения, логического сложения, последними выполняются операции импликации и эквивалентности).

В логическом выражении могут использоваться скобки, которые изменяют порядок выполнения операций.

Пример. Найти значение выражения:

$1 ≤ a ∨ A ∨ sin(π/a — π/b) < 1 ∧ ¬B ∧ ¬(b^a + a^b > a + b ∨ A ∧ B)$ для а = 2, b = 3, A = истина, В = ложь.

Решение. Порядок подсчета значений:

1) ba + ab > a + b, после подстановки получим: 32 + 23 > 2 + 3, т. е. 17 > 2 + 3 = истина;

2) A ∧ B = истина ∧ ложь = ложь.

Следовательно, выражение в скобках равно (ba + ab > a + b ∨ A ∧ B) = истина ∨ ложь = истина;

3) 1≤ a = 1 ≤ 2 = истина;

4) sin(π/a — π/b)  < 1 = sin(π/2 — π/3) < 1 = истина.

После этих вычислений окончательно получим: истина ∨ А ∧ истина ∧ ¬В ∧ ¬истина.

Теперь должны быть выполнены операции отрицания, затем логического умножения и сложения:

5) ¬В = ¬ложь = истина; ¬истина = ложь;

6) A ∧ истина ∧ истина ∧ ложь = истина ∧ истина ∧ истина ∧ ложь = ложь;

7) истина ∨ ложь = истина.

Таким образом, результат логического выражения при заданных значениях— «истина».

Примечание. Учитывая, что исходное выражение есть, в конечном итоге, сумма двух слагаемых, и значение одного из них 1 ≤ a = 1 ≤ 2 = истина, без дальнейших вычислений можно сказать, что результат для всего выражения тоже «истина».

Тождественные преобразования логических выражений

В алгебре логики выполняются основные законы, позволяющие производить тождественные преобразования логических выражений.

Закон Для ∨ Для ∧
Переместительный A ∨ B = B ∨ A A ∧ B = B ∧ A
Сочетательный A ∨ (B ∨ C) = (B ∨ A) ∨ C A ∧ (B ∧ C) = (A ∧ B) ∧ C
Распределительный A ∧ (B ∨ C) = (A ∧ B) ∨ (A ∧ C) A ∨ B ∧ C = (A ∨ B) ∧ (A ∨ C)
Правила де Моргана ${A ∨ B}↖{-}$ = $A↖{-} ∧ B↖{-}$ ${A ∧ B}↖{-}$ = $A↖{-} ∨ B↖{-}$
Идемпотенции A ∨ A = A A ∧ A = A
Поглощения A ∨ A ∧ B = A A ∧ (A ∨ B) = A
Склеивания (A ∧ B) ∨ (A↖{-} ∧ B) = B (A ∨ B) ∧ (A↖{-} ∨ B) = B
Операция переменной с ее инверсией $A ∨ A↖{-}$ = 1 $A ∧ A↖{-}$ = 0
Операция с константами A ∨ 0 = A
A ∨ 1 = 1
A ∧ 1 = A
A ∧ 0 = 0
Двойного отрицания $A↖{=}$ = A

Доказательства этих утверждений производят на основании построения таблиц истинности для соответствующих записей.

Равносильные преобразования логических формул имеют то же назначение, что и преобразования формул в обычной алгебре. Они служат для упрощения формул или приведения их к определенному виду путем использования основных законов алгебры логики. Под упрощением формулы, не содержащей операций импликации и эквивалентности, понимают равносильное преобразование, приводящее к формуле, которая содержит либо меньшее по сравнению с исходной число операций, либо меньшее число переменных.

Некоторые преобразования логических формул похожи на преобразования формул в обычной алгебре (вынесение общего множителя за скобки, использование переместительного и сочетательного законов и т. п.), тогда как другие преобразования основаны на свойствах, которыми не обладают операции обычной алгебры (использование распределительного закона для конъюнкции, законов поглощения, склеивания, де Моргана и др.).

Рассмотрим на примерах некоторые приемы и способы, применяемые при упрощении логических формул:

1) X1 ∧ X2 ∨ X1 ∧ X2 ∪ ¬X1 ∧ X2 = X1 ∧ X2 ∨ ¬X1 ∧ X2 = (X1 ∨ ¬X1) ∧ X2 = 1 ∧ X2 = X2 .

Для преобразования здесь можно применить закон идемпотенции, распределительный закон; операцию переменной с инверсией и операцию с константой.

2) X1 ∨ X1 ∧ X2 = X1 ∨ (1 ∨ 1 ∧ X2) = X1 ∨ (1 ∨ X2) = X1 .

Здесь для упрощения применяется закон поглощения.

3) ¬(X1 ∧ X2) ∨ X2 = (¬X1 ∨ ¬X2) ∨ X2 = ¬X1 ∨ ¬X2 ∨ X2 = ¬X1 ∨ 1 = 1 .

При преобразовании применяются правило де Моргана, операция переменной с ее инверсией, операция с константой

Примеры решения задач

Пример 1. Найти логическое выражение, равносильное выражению A ∧ ¬(¬B ∨ C) .

Решение. Применяем правило де Моргана для В и С: ¬(¬B ∨ C) = B ∧ ¬C .

Получаем выражение, равносильное исходному: A ∧ ¬(¬B ∨ C) = A ∧ B ∧ ¬C .

Ответ: A ∧ B ∧ ¬C.

Пример 2. Указать значение логических переменных А, В, С, для которых значение логического выражения (A ∨ B) → (B ∨ ¬C ∨ B) ложно.

Решение. Операция импликации ложна только в случае, когд а из истинной посылки следует ложь. Следовательно, для заданного выражения посылка A ∨ B должна принимать значение «истина», а следствие, т. е. выражение B ∨ ¬C ∨ B , — «ложь».

1) A ∨ B — результат дизъюнкции — «истина», если хотя бы один из операндов — «истина»;

2) B ∨ ¬C ∨ B — выражение ложно, если все слагаемые имеют значение «ложь», т. е. В — «ложь»; ¬C — «ложь», а следовательно, переменная С имеет значение «истина»;

3) если рассмотреть посылку и учесть, что В — «ложь», то получим, что значение А — «истина».

Ответ: А — истина, В — ложь, С — истина.

Пример 3. Каково наибольшее целое число X, при котором истинно высказывание (35 < X · X) → (X < (X — 3)) ?

Решение. Запишем таблицу истинности для операции импликации:

A B A → B
1 0 0
0 1 1
0 0 1
1 1 1

Выражение X < (X — 3) ложно при любых положительных значениях X. Следовательно, для того чтобы результатом импликации была «истина», необходимо и достаточно, чтобы выражение 35 < X · X также было ложно. Максимальное целое значение X, для которого 35 < X · X ложно, равно 5.

Ответ: X = 5.

Использование логических выражений для описания геометрических областей

Логические выражения могут быть использованы для описания геометрических областей. В этом случае задача формулируется так: записать для заданной геометрической области такое логическое выражение, которое принимает значение «истина» для значений x, y тогда и только тогда, когда любая точка с координатами (x; y) принадлежит геометрической области.

Рассмотрим описание геометрической области с помощью логического выражения на примерах.

Пример 1. Задано изображение геометрической области. Записать логическое выражение, описывающее множество точек, принадлежащих ей.

1) .

Решение. Заданную геометрическую область можно представить в виде набора следующих областей: первая область — D1 — полуплоскость ${x}/{-1} +{y}/{1} ≤ 1$, вторая — D2 — круг с центром в начале координат $x^2 + y^2 ≤ 1$. Их пересечение D1 $∩$ D2 представляет собой искомую область.

Результат: логическое выражение ${x}/{-1}+{y}/{1} ≤ 1 ∧ x^2 + y^2 ≤ 1$.

2)

Эту область можно записать так: |x| ≤ 1 ∧ y ≤ 0 ∧ y ≥ -1 .

Примечание. При построении логического выражения используются нестрогие неравенства, а это значит, что границы фигур также принадлежат заштрихованной области. Если использовать строгие неравенства, то границы учитываться не будут. Границы, не принадлежащие области, обычно изображаются пунктиром.

Можно решить обратную задачу, а именно: нарисовать область для заданного логического выражнения.

Пример 2. Нарисовать и заштриховать область, для точек которой выполняется логическое условие y ≥ x ∧ y + x ≥ 0 ∧ y < 2 .

Решение. Искомая область представляет собой пересечение трех полуплоскостей. Строим на плоскости (x, y) прямые y = x; y = –x; y = 2. Это границы области, причем последняя граница y = 2 не принадлежит области, поэтому ее наносим пунктирной линией. Для выполнения неравенства y ≥ x нужно, чтобы точки находились слева от прямой y = x, а неравенство y = –x выполняется для точек, которые находятся справа от прямой y = –x. Условие y < 2 выполняется для точек, лежащих ниже прямой y = 2. В результате получим область, которая изображена на рис.:

Использование логических функций для описания электрических схем

Логические функции очень удобны для описания работы электрических схем. Так, для схемы, представленной на рис., где значение переменной X — это состояние выключателя (если он включен, значение X — «истина», а если выключен — «ложь»), это значение Y — это состояние лампочки (если она горит — значение «истина», а если нет — «ложь»), логическая функция запишется так: Y = X . Функцию Y называют функцией проводимости.

Для схемы, представленной на рис., логическая функция Y имеет вид: Y = X1 ∪ X2, т. к. достаточно одного включенного выключателя, чтобы горела лампочка. В схеме на рис., для того чтобы горела лампочка, должны быть включены оба выключателя, следовательно, функция проводимости имеет вид: Y = X1 ∧ X2 .

Для более сложной схемы функция проводимости будет иметь вид: Y = (X11 ∨ (X12 ∧ X13)) ∧ X2 ∧ (X31 ∨ X32).

Схема также может содержать контакты на замыкание. В этом случае размыкаемый контакт как выключатель обеспечивает загорание лампочки, когда кнопка отпущена, а не нажата. Для таких схем размыкающий выключатель описывается отрицанием.

Две схемы называются равносильными, если через одну из них ток проходит тогда, когда он проходит и через другую. Из двух равносильных схем более простой считается схема, функция проводимости которой содержит меньшее число элементов. Задача нахождения наиболее простых схем среди равносильных очень важна.

Использование аппарата алгебры логики при проектировании логических схем

Математический аппарат алгебры логики очень удобен для описания того, как функционируют аппаратные средства компьютера. Любая информация при обработке на компьютере представляется в двоичной форме, т. е. кодируется некоторой последовательностью 0 и 1. Обработку двоичных сигналов, соответствующих 0 и 1, выполняют в компьютере логические элементы. Логические элементы, которые выполняют основные логические операции И, ИЛИ, НЕ, представлены на рис.

Условные обозначения логических элементов являются стандартными и используются при составлении логических схем компьютера. С помощью этих схем можно реализовать любую логическую функцию, описывающую работу компьютера.

Технически компьютерный логический элемент реализуется в виде электрической схемы, которая представляет собой соединение различных деталей: диодов, транзисторов, резисторов, конденсаторов. На вход логического элемента, который называют также вентилем, поступают электрические сигналы высокого и низкого уровней напряжения, на выход выдается один выходной сигнал также либо высокого, либо низкого уровня. Эти уровни соответствуют одному из состояний двоичной системы: 1 — 0; ИСТИНА — ЛОЖЬ. Каждый логический элемент имеет свое условное обозначение, которое выражает его логическую функцию, но не указывает на то, какая именно электронная схема в нем реализована. Это упрощает запись и понимание сложных логических схем. Работу логических схем описывают с помощью таблиц истинности. Условное обозначение на схеме ИЛИ знак «1» — от устаревшего обозначения дизъюнкции как «>=1» (значение дизъюнкции равно 1, если сумма двух операндов больше или равна 1). Знак «&» на схеме И является сокращенной записью английского слова and.

Из логических элементов составляются электронные логические схемы, выполняющие более сложные логические операции. Набор логических элементов, состоящий из элементов НЕ, ИЛИ, И, с помощью которых можно построить логическую структуру любой сложности, называется функционально полным.

Построение таблиц истинности логических выражений

Для логической формулы всегда можно записать таблицу истинности, т. е. представить заданную логическую функцию в табличном виде. В этом случае таблица должна содержать все возможные комбинации аргументов функции (формулы) и соответствующие значения функции (результаты формулы на заданном наборе значений).

Удобной формой записи при нахождении значений функции является таблица, содержащая, кроме значений переменных и значений функции, также значения промежуточных вычислений. Рассмотрим пример построения таблицы истинности для формулы ${X1}↖{-} ∧ X2 ∨ {X1 ∨ X2}↖{-} ∨ X1$.

X1 X2 ${X1}↖{-}$ ${X1}↖{-}$ X2 X1 ∧ X2 ${X1 ∨ X2}↖{-}$ ${X1}↖{-}$ ∧ X2 ∨ ${X1 ∨ X2}↖{-}$ ${X1}↖{-}$ ∧ X2 ∨ ${X1 ∨ X2}↖{-}$ ∨ X1
1 1 0 0 1 0 0 1
1 0 0 0 1 0 0 1
0 1 1 1 1 0 1 1
0 0 1 0 0 1 1 1

Если функция принимает значение 1 при всех наборах значений переменных, она является тождественно-истинной; если при всех наборах входных значений функция принимает значение 0, она является тождественно-ложной; если набор выходных значений содержит как 0, так и 1, функция называется выполнимой. Приведенный выше пример является примером тождественно-истинной функции.

Зная аналитическую форму логической функции, всегда можно перейти к табличной форме логических функций. С помощью заданной таблицы истинности можно решить обратную задачу, а именно: для заданной таблицы построить аналитическую формулу логической функции. Различают две формы построения аналитической зависимости логической функции по таблично заданной функции.

1. Дизъюнктивно нормальная форма (ДНФ) — сумма произведений, образованных из переменных и их отрицаний для ложных значений.

Алгоритм построения ДНФ следующий:

  1. в таблице истинности функции выбирают наборы аргументов, для которых логические формы равны 1 («истина»);
  2. все выбранные логические наборы как логические произведения аргументов записывают, последовательно соединив их между собой операцией логической суммы (дизъюнкции);
  3. для аргументов, которые являются ложными, в построенной записи проставляют операцию отрицания.

Пример. Построить функцию, определяющую, что первое число равно второму, используя метод ДНФ. Таблица истинности функции имеет вид

X1 X2 F(X1, X2)
1 1 1
0 1 0
1 0 0
0 0 1

Решение. Выбираем наборы значений аргументов, в которых функция равна 1. Это первая и четвертая строки таблицы (строку заголовка при нумерации не учитываем).

Записываем логические произведения аргументов этих наборов, объединив их логической суммой: X1 ∧ X2 ∨ X1 ∧ X2 .

Записываем отрицание относительно аргументов выбранных наборов, имеющих ложное значение (четвертая строка таблицы; второй набор в формуле; первый и второй элементы): X1 ∧ X2 ∨ ${X1}↖{-}$ ∧ ${X2}↖{-}$.

Ответ: F(X1, X2) = X1 ∧ X2 ∨ ${X1}↖{-}$ ∧ ${X2}↖{-}$.

2. Конъюнктивно нормальная форма (КНФ) — произведение сумм, образованных из переменных и их отрицаний для истинных значений.

Алгоритм построения КНФ следующий:

  1. в таблице истинности выбирают наборы аргументов, для которых логические формы равны 0 («ложь»);
  2. все выбранные логические наборы как логические суммы аргументов записывают последовательно, соединив их между собой операцией логического произведения (конъюнкции);
  3. для аргументов, которые являются истинными, в построенной записи проставляют операцию отрицания.

Примеры решения задач

Пример 1. Рассмотрим предыдущий пример, т. е. построим функцию, определяющую, что первое число равно второму, используя метод КНФ. Для заданной функции ее таблица истинности имеет вид

X1 X2 F(X1, X2)
1 1 1
0 1 0
1 0 0
0 0 1

Решение. Выбираем наборы значений аргументов, в которых функция равна 0. Это вторая и третья строки (строку заголовка при нумерации не учитываем).

Записываем логические суммы аргументов этих наборов, объединив их логическим произведением: X1 ∨ X2 ∧ X1 ∨ X2 .

Записываем отрицание относительно аргументов выбранных наборов, имеющих истинное значение (вторая строка таблицы, первый набор формулы, второй элемент; для третьей строки, а это второй набор формулы, первый элемент): X1 ∨ ${X2}↖{-}$ ∧ ${X1}↖{-}$ ∨ X2.

Таким образом, получена запись логической функции в КНФ.

Ответ: X1 ∨ ${X2}↖{-}$ ∧ ${X1}↖{-}$ ∨ X2.

Полученные двумя методами значения функций являются эквивалентными. Для доказательства этого утверждения используем правила логики: F(X1, X2) = X1 ∨ ${X2}↖{-}$ ∧ ${X1}↖{-}$ ∨ X2 = X1 ∧ ${X1}↖{-}$ ∨ X1 ∧ X2 ∨ ${X2}↖{-}$ ∧ ${X1}↖{-}$ ∨ ${X2}↖{-}$ ∧ X2 = 0 ∨ X1 ∨ X2 ∨ ${X2}↖{-}$ ∧ ${X1}↖{-}$ ∨ 0 = X1 ∧ X2 ∨ ${X1}↖{-}$ ∧ ${X2}↖{-}$.

Пример 2. Построить логическую функцию для заданной таблицы истинности:

X1 X2 F(X1, X2)
1 1 1
1 0 0
0 1 1
0 0 0

Решение. Используем алгоритм ДНФ для построения исходной функции:

X1 X2 F(X1, X2)    
1 1 1 X1 ∧ X2
1 0 0    
0 1 1 ${X1}↖{-}$ ∧ X2
0 0 0    

Искомая формула: X1 ∧ X2 ∨ ${X1}↖{-}$ ∧ X2 .

Ее можно упростить: X1 ∧ X2 ∨ ${X1}↖{-}$ ∧ X2 = X2 ∧ (X1 ∨ ${X1}↖{-}$) = X2 ∧ 1 = X2.

Пример 3. Для приведенной таблицы истинности построить логическую функцию, используя метод ДНФ.

X1 X2 X3 F(X1, X2, X3)    
1 1 1 1 X1 ∧ X2 ∧ X3
1 0 1 0    
0 1 1 1 ${X1}↖{-}$ ∧ X2 ∧ X3
0 0 1 0    
1 1 0 1 X1 ∧ X2 ∧ ${X3}↖{-}$
1 0 0 1 X1 ∧ ${X2}↖{-}$ ∧ ${X3}↖{-}$
0 1 0 0    
0 0 0 0    

Искомая формула: X1 ∧ X2 ∧ X ∨ ${X1}↖{-}$ ∧ X2 ∧ X3 ∨ X1 ∧ X2 ∧ ${X3}↖{-}$ ∪ X1 ∧ ${X2}↖{-}$ ∧ ${X3}↖{-}$.

Формула достаточно громоздка, и ее следует упростить:

X1 ∧ X2 ∧ X3 ∨ ${X1}↖{-}$ ∧ X2 ∧ X3 ∨ X1 ∧ X2 ∧ ${X3}↖{-}$ ∨ X1 ∧ ${X2}↖{-}$ ∧ ${X3}↖{-}$ = X2 ∧ X3 ∧ (X1 ∨ ${X1}↖{-}$) ∨ X1 ∧ ${X3}↖{-}$ ∧ (X2 ∨ ${X2}↖{-}$) = X2 ∧ X3 ∨ X1 ∧ ${X3}↖{-}$.

Таблицы истинности для решения логических задач

Составление таблиц истинности — один из способов решения логических задач. При использовании такого способа решения, условия, которые содержит задача, фиксируются с помощью специально составленных таблиц.

Примеры решения задач

Пример 1. Составить таблицу истинности для охранного устройства, которое использует три датчика и срабатывает при замыкании только двух из них.

Решение. Очевидно, что результатом решения будет таблица, в которой искомая функция Y(X1, X2, X3) будет иметь значение «истина», если какие-либо две переменные имеют значение «истина».

X1 X2 X3 Y(X1, X2, X3)
1 1 1 0
1 1 0 1
1 0 1 1
1 0 0 0
0 1 1 1
0 1 0 0
0 0 1 0
0 0 0 0

Пример 2. Составить расписание уроков на день, учитывая, что урок информатики может быть только первым или вторым, урок математики — первым или третьим, а физики — вторым или третьим. Возможно ли составить расписание, удовлетворив всем требованиям? Сколько существует вариантов расписания?

Решение. Задача легко решается, если составить соответствующую таблицу:

  1-й урок 2-й урок 3-й урок
Информатика 1 1 0
Математика 1 0 1
Физика 0 1 1

Из таблицы видно, что существуют два варианта искомого расписания:

  1. математика, информатика, физика;
  2. информатика, физика, математика.

Пример 3. В спортивный лагерь приехали трое друзей — Петр, Борис и Алексей. Каждый из них увлекается двумя видами спорта. Известно, что таких видов спорта шесть: футбол, хоккей, лыжи, плавание, теннис, бадминтон. Также известно, что:

  1. Борис — самый старший;
  2. играющий в футбол младше играющего в хоккей;
  3. играющие в футбол и хоккей и Петр живут в одном доме;
  4. когда между лыжником и теннисистом возникает ссора, Борис мирит их;
  5. Петр не умеет играть ни в теннис, ни в бадминтон.

Какими видами спорта увлекается каждый из мальчиков?

Решение. Составим таблицу и отразим в ней условия задачи, заполнив соответствующие клетки цифрами 0 и 1 в зависимости от того, ложно или истинно соответствующее высказывание.

Так как видов спорта шесть, получается, что все мальчики увлекаются разными видами спорта.

Из условия 4 следует, что Борис не увлекается ни лыжами, ни теннисом, а из условий 3 и 5, что Петр не умеет играть в футбол, хоккей, теннис и бадминтон. Следовательно, любимые виды спорта Петра — лыжи и плавание. Занесем это в таблицу, а оставшиеся клетки столбцов «Лыжи» и «Плавание» заполним нулями.

  Футбол Хоккей Лыжи Плавание Бадминтон Теннис
Петр 0 0 1 1 0 0
Борис     0 0   0
Алексей     0 0    

Из таблицы видно, что в теннис может играть только Алексей.

Из условий 1 и 2 следует, что Борис не футболист. Таким образом, в футбол играет Алексей. Продолжим заполнять таблицу. Внесем в пустые ячейки строки «Алексей» нули.

  Футбол Хоккей Лыжи Плавание Бадминтон Теннис
Петр 0 0 1 1 0 0
Борис 0   0 0   0
Алексей 1 0 0 0 0 1

Окончательно получаем, что Борис увлекается хоккеем и бадминтоном. Итоговая таблица будет выглядеть следующим образом:

  Футбол Хоккей Лыжи Плавание Бадминтон Теннис
Петр 0 0 1 1 0 0
Борис 0 1 0 0 1 0
Алексей 1 0 0 0 0 1

Ответ: Петр увлекается лыжами и плаванием, Борис играет в хоккей и бадминтон, а Алексей занимается футболом и теннисом.

Логические значения, операции, выражения

Код ОГЭ: 1.3.3. Логические значения, операции, выражения

Алгебра логики, логические высказывания

Наука, изучающая формы, методы и законы правильного мышления, называется логикой. Она интересуется не содержанием мышления, а его формой, поэтому ее часто называют еще формальной логикой.

Форма мышления — это способ выражения мыслей или форма, по которой они строятся.

Форма, обозначающая какой–либо объект или отличающий его признак, называется понятием. Примеры понятий: «компьютер», «планета», «длина», «профессия».

Форма, утверждающая или отрицающая что–либо о свойствах понятий и отношений между ними, называется утверждением (высказыванием, суждением). Примеры логических утверждений: «Декодирование — процесс восстановления информации из закодированного представления»; «В двоичной системе используются две цифры: 0 и 1»; «Париж — столица Франции». Утверждения могут быть истинными или ложными. Так, высказывание «Шанхай — столица Франции» является ложным утверждением.

Форма, в которой из двух или нескольких высказываний получают новое утверждение, называется умозаключением. Пример умозаключения: «Периферийные устройства компьютера — это устройства для ввода или вывода информации. Сканер — устройство для переноса текста и изображений с бумаги в компьютер. Следовательно, сканер — периферийное устройство».

Правила, которые должны соблюдаться, чтобы на основании истинных суждений получить истинные выводы, — это законы мышления. Логика изучает эти законы и способы получения новых утверждений на основании уже имеющихся.

Математическая логика использует для установления истинности или ложности высказываний математические методы. Она пользуется специальным символьным языком, подобным языку математики, поэтому ее часто называют символьной логикой.

Алгебра логики — раздел математической логики, в котором методы алгебры используются в логических преобразованиях. Она изучает логические высказывания и методы установления их истинности или ложности с помощью алгебраических методов.

Логическое высказывание — это любое повествовательное предложение, в отношении которого можно однозначно утверждать, что его содержание истинно или ложно. Вопросительные и повелительные предложения не являются логическими высказываниями. Но и не каждое повествовательное предложение является логическим высказыванием. Например, суждение «Лето было очень дождливым» не является однозначным, для утверждения «Существует несколько Вселенных» нельзя однозначно определить истинность; поэтому такие предложения не являются логическими высказываниями (утверждениями).

Таким образом, отличительной особенностью логических высказываний является возможность принимать одно из двух значений — истина и ложь. Истинность или ложность высказывания определяется вне алгебры логики — с помощью наблюдений, научных исследований, практических опытов и т. п.

В алгебре логики различают простые высказывания и сложные (составные), составленные из нескольких простых. Если в высказывании нельзя выделить некую часть, которая не совпадает по смыслу с исходным высказыванием и сама является высказыванием, то оно называется простым высказыванием. Простые высказывания обычно обозначаются латинскими буквами A, B, C и т. д.

Сложные высказывания представляют собой объединение простых высказываний с помощью логических связок. В качестве логических связок используются слова «не», «и», «или», «тогда и только тогда», «если … то». Истинность или ложность получаемых таким образом сложных высказываний определяется значением простых высказываний. Например, из простых высказываний «Офис фирмы находится в Мадриде» и «Офис фирмы находится в Берлине» можно составить сложные: «Офис фирмы находится в Мадриде или Берлине», «Офис фирмы находится в Мадриде и Берлине», «Если офис фирмы находится в Мадриде, то он находится в Берлине». Истинность первого из них означает, что офис фирмы находится в одном из названных городов или же имеются офисы в обоих городах. Ложность его означает, что ни в одном из этих городов офиса нет. Второе составное утверждение истинно тогда, когда в обоих городах имеется офис фирмы. Если же офис существует только в Берлине или только в Мадриде, — второе составное высказывание ложно.

В классической, двузначной алгебре логики логических значений всего два: истина (True) и ложь (False). Им соответствует цифровое представление — 1 и 0. Иногда эти значения записывают как «да» и «нет». Факт истинности или ложности некоторого высказывания А записывают соответственно как А = 1 или А = 0.

Логические операции

В алгебре логики логические связки рассматриваются как логические операции. Они имеют свои названия и обозначения. Результаты применения каждой операции к логическим высказываниям (истинным или ложным) можно представить в виде таблицы. В ней указывают все возможные сочетания значений исходных логических высказываний и истинность или ложность результата. Такие таблицы называют таблицами истинности операции. Обычно в них используют обозначения логических значений 0 и 1 или ложь и истина.

Основные логические операции — отрицание, конъюнкция, дизъюнкция, исключающая дизъюнкция, следование, эквивалентность.

Логическое отрицание (инверсия) — логическая операция, в результате которой из данного высказывания получается новое высказывание — отрицание исходного. Обозначается символически чертой сверху (Ā) или условными обозначениями ¬А, not А, не А (читается «отрицание А», «не А», «А ложно», «неверно, что А»).

Высказывание ¬А ложно, когда А истинно, и истинно, когда А ложно.

Таблица истинности операции отрицания

Если обозначить через А высказывание «Арбуз является ягодой», то ¬А соответствует высказыванию «Арбуз не является ягодой» («Неверно, что арбуз — ягода»).

Отрицание является унарной операцией. Унарная (одноместная) операция — это операция, которая применяется к одному операнду.

Остальные логические операции являются двуместными (бинарными). Бинарная (двуместная) операция — это операция, которая выполняется над двумя операндами.

Логическое умножение (конъюнкция) — операция, соединяющая два или более высказываний при помощи связки «и». Эта связка символически обозначается с помощью знака ∧ и читается «А и В». Для обозначения конъюнкции также применяются знаки: А • В, А & В, А и В, А and В, а иногда между высказываниями не ставится никакого знака: АВ.

Высказывание А ∧ В истинно только тогда, когда оба высказывания А и В истинны. Высказывание А ∧ В ложно только тогда, когда ложно хотя бы одно из высказываний А или В.

Таблица истинности операции конъюнкции

Таблица истинности операции конъюнкции

Например, высказывания «Лондон расположен севернее Лиссабона» и «Лондон расположен восточнее Лиссабона» истинны. Тогда истинным будет и составное логическое высказывание «Лондон расположен севернее и восточнее Лиссабона». Высказывания «Лондон расположен не севернее и восточнее Лиссабона», «Лондон расположен севернее и не восточнее Лиссабона», «Лондон расположен не севернее и не восточнее Лиссабона» — ложны.

Логическое сложение (дизъюнкция) — операция, соединяющая два или более высказываний при помощи связки « или». Эта связка символически обозначается с помощью знака v и читается «А или В». Для обозначения дизъюнкции также применяются знаки: А + В, А или В, А or В, А | B.

Высказывание А v В истинно только тогда, когда хотя бы одно из высказываний А или В истинно. Высказывание А v В ложно только тогда, когда оба высказывания А и В ложны.

Таблица истинности операции дизъюнкции

Таблица истинности операции дизъюнкции

Например, высказывания «Виктор старше Ольги» и «Виктор — однофамилец Ольги» истинны. Тогда истинными будут и составные логические высказывания «Виктор старше Ольги или Виктор — однофамилец Ольги», «Виктор младше Ольги или Виктор — однофамилец Ольги», «Виктор старше Ольги или Виктор — не однофамилец Ольги». Высказывание «Виктор младше Ольги или Виктор — не однофамилец Ольги» — ложно, поскольку ложны оба составляющие его простые высказывания.

Исключающее сложение (исключающая дизъюнкция, строгая дизъюнкция, сложение по модулю два, дизъюнкция строго–разделительная) — логическая операция, соединяющая два высказывания при помощи связки «или», употребленной в исключающем смысле (называется также исключающее «или»). Операция символически обозначается с помощью знака ⊕ и читается «либо А, либо В».

Высказывание А ⊕ В истинно только тогда, когда высказывания А и В имеют различные значения.

Таблица истинности операции строгой дизъюнкции

Таблица истинности операции строгой дизъюнкции

Например, результат исключающей дизъюнкции двух высказываний «Виктор не старше Ольги» и «Виктор младше Ольги» всегда будет истиной, кто бы из них не был старше.

Логическое следование (импликация) — логическая операция, соединяющая два высказывания при помощи связки «если… то» в сложное высказывание. Операция символически обозначается с помощью знака → и читается «Если А, то В», «А влечет В», «из А следует В», «А имплицирует В». Для обозначения импликации применяются также знаки ⊃ или ⇒. Первое логическое высказывание является условием (посылкой), а второе — следствием (заключением).

Для операции импликации справедливо утверждение, что из лжи может следовать все что угодно, а из истины — только истина. Таким образом, импликация А → В ложна только тогда, когда А истинно, а В ложно (из истинного высказывания следует ложное). Во всех остальных случаях импликация истинна.

Таблица истинности операции импликации

Таблица истинности операции импликации

Для высказываний «Луна — спутник Земли» и «Сумма углов треугольника не равна 180°» (первое истинно, второе ложно) составное высказывание «Если Луна — спутник Земли, то сумма углов треугольника не равна 180°» будет ложным. Однако истинными будут высказывания «Если Луна — спутник Земли, то сумма углов треугольника равна 180°», «Если Луна — не спутник Земли, то сумма углов треугольника не равна 180°» и «Если Луна — не спутник Земли, то сумма углов треугольника равна 180°». Этот пример наглядно демонстрирует, что в алгебре логики смысл высказываний не учитывается, а рассматриваются только их истинность или ложность.

Логическое равенство (эквивалентность, следование, двойная импликация, равнозначность) — логическая операция, позволяющая из двух высказываний А и В получить новое высказывание А ≡ В (читается «А эквивалентно B»). Эта операция может быть выражена связками «тогда и только тогда», «необходимо и достаточно», «равносильно». Для обозначения эквивалентности применяются знаки ~, ⇔.

Если оба высказывания имеют различные логические значения, результатом операции эквивалентности всегда будет ложь. Если же оба простые высказывания ложны или оба истинны, то составное логическое высказывание всегда будет истинно.

Таблица истинности операции эквивалентности

Таблица истинности операции эквивалентности

Для высказываний «Линейное уравнение всегда имеет решение» и «Кит — млекопитающее» их эквивалентность всегда будет истиной, так как оба простые утверждения истинны.

Таким образом, сводная таблица истинности для всех основных логических операций имеет вид:

Логические выражения

Логические высказывания могут быть записаны в виде формул (логических выражений). Логические выражения включают логические переменные, знаки логических операций, логические константы (истина и ложь) и скобки. Логические выражения принимают значения истина или ложь.

Правила построения логических выражений:

  • любая логическая переменная или константа (истина и ложь) являются логическим выражением;
  • если А — , то ¬А — тоже логическое выражение;
  • если А и В — логические выражения, то А ∧ В; А v В ; А ⊕ В; А → В; А ~ В — тоже логические выражения.

Например, A ⊕ истина v В v ложь — логическое выражение; А v ⊕ В v ложь не является логическим выражением.

Логическое выражение, принимающее значение истина при любых значениях входящих в него переменных, называется тождественно–истинным выражением (тавтологией). Например, А v В v ¬А; (А ∧ ¬А) → В.

Логическое выражение, принимающее значение ложь при любых значениях входящих в него переменных, называется тождественно–ложным выражением (противоречием). Например, А ∧ ¬А; В ~ ¬В.

Логическое выражение, принимающее как значение ложь, так и значение истина при разных значениях входящих в него переменных, называется выполнимым выражением.

ПРИОРИТЕТ ЛОГИЧЕСКИХ ОПЕРАЦИЙ

Для сложных логических выражений, содержащих несколько логических операций, определен порядок выполнения действий (приоритет): сначала операции отрицания, затем операции логического умножения, потом операции логического сложения и исключающего сложения, последними выполняются операции импликации и эквивалентности. Операции выполняются слева направо. Порядок выполнения может быть изменен с помощью скобок.

Приоритет выполнения логических операций в логических выражениях

Приоритет выполнения логических операций в логических выражениях

Пример 1
Определить порядок выполнения логических операций в выражении.
Вычислить его значение, если А = 1, В = 0, С = 1.
¬А ∧ С v (A ⊕ В) ∧ В

Решение. Первыми вычисляются значения в скобках. Затем выполняются операции по приоритетам: самый высокий приоритет имеет операция отрицания, после нее, как в математике, следуют операции умножения, а затем сложения. Таким образом, порядок будет следующий:

Тогда значение выражения ¬1 ∧ 1 v (1 ⊕ 0) ∧ 0 после вычисления отрицания и выражения в скобках: 0 ∧ 1 v 1 ∧ 0, после операций умножения: 0 v 0. Итог: 0.
Ответ: логическое выражение ложно.

Пример 2
Для каких из приведенных слов истинно следующее высказывание?
(Вторая буква гласная) ИЛИ (Первая буква гласная) И НЕ (Длина имени не больше 5 букв)

1) Олег   2) Марианна  3) Светлана   4) Ольга.

Решение.

  1.  В соответствии с приоритетом выполнения операций сначала нужно вычислить результат операции отрицания: «Длина имени больше 5 букв». Такие имена — Марианна, Светлана.
  2. Затем необходимо рассчитать результат конъюнкции (связка И) второго и третьего высказываний. Конъюнкция истинна только тогда, когда истинны оба высказывания. Следовательно, нужно выбрать имена, в которых и первая буква гласная, и длина имени больше 5 букв — таких имен среди вариантов нет.
  3. Результат дизъюнкции (связка ИЛИ) истинен только тогда, когда истинно или одно, или второе, или одновременно оба простые высказывания. Первое простое высказывание истинно для имени Марианна, второе ложно для указанных имен. Таким образом, верный вариант ответа — 2.

Ответ: 2) Марианна.

СВЯЗЬ МЕЖДУ ЛОГИЧЕСКИМИ ОПЕРАЦИЯМИ

Между логическими операциями существует взаимосвязь. Операции исключающего «или», следования и эквивалентности можно выразить через операции отрицания, логического сложения и умножения, что отражено в следующей таблице.

Связь между логическими операциями

Поэтому операции отрицания, логического сложения и логического умножения называют основными логическими операциями: их достаточно, чтобы построить любое логическое выражение.

Если логическое выражение содержит только операции отрицания, логического сложения и логического умножения, говорят, что выражение находится в нормальной форме.

РАВНОСИЛЬНЫЕ ЛОГИЧЕСКИЕ ВЫРАЖЕНИЯ

Логические выражения, значения которых совпадают для всех наборов входящих в них переменных, называются равносильными, или эквивалентными.

Чтобы убедиться, что два выражения равносильны, можно построить для них таблицы истинности. Если в таблицах совпадут все значения, значит, выражения равносильны.

Пример 3
Проверить равносильность выражений А ~ E и (Ā ∧ Ē) v (A ∧ E).

Решение. Для проверки следует создать таблицу истинности, содержащую столько строк, сколько возможно наборов значений переменных, входящих в выражение. Для двух переменных (А и E) количество наборов равно четырем. К двум столбцам для значений переменных (А и E) нужно присовокупить количество столбцов, равное количеству операций в выражении. Таким образом, необходимо создать таблицу, содержащую 4 строки и 7 столбцов.

Заполним первые 2 столбца (А и E) всеми сочетаниями значений переменных. Запишем в качестве заголовков столбцов все операции выражения в порядке их выполнения (в соответствии с приоритетами и скобками). Рассчитаем значения этих операций: сначала выражения в скобках, затем результат их сложения.

Последний столбец содержит результирующее значение выражения. Он совпадает с таблицей истинности для операции эквивалентности. Следовательно, выражения равносильны.

Основные законы алгебры логики

Для сложных логических выражений с большим числом переменных определение их истинности путем построения таблиц истинности становится громоздким. В таких случаях применяют способы упрощения выражений. Под упрощением понимают равносильное преобразование выражения к его нормальной форме.

Нормальная форма выражения содержит только операции отрицания, конъюнкции и дизъюнкции и не содержит отрицания выражений и двойных отрицаний.

Для упрощения используют равносильные преобразования, которые иначе называют основными законами алгебры логики.

Тождественные преобразования логических выражений

Тождественные преобразования логических выражений

Для всех тождественных преобразований выполняется закон двойственности: если в формуле преобразования заменить конъюнкцию на дизъюнкцию, дизъюнкцию — на конъюнкцию, значения 1 — на 0, 0 — на 1, то закон, сформулированный для конъюнкции, примет форму аналогичного закона для дизъюнкции, и наоборот.

Прежде всего при равносильных преобразованиях избавляются от отрицания выражений, потом — от логических операций исключающей дизъюнкции, следования и эквивалентности. Затем используют законы алгебры логики для уменьшения количества переменных в выражении.

Пример 4
Выбрать выражение, которое равносильно выражению (A ∧ B) v (Ā ∧ B).

1) A         2) A ∧ B          3) Ā ∧ B           4) B

Решение. В соответствии с законом склеивания (A ∧ B) v (Ā ∧ B) = B, следовательно, исходное выражение равносильно выражению В.
Ответ: 4) В.

ОПРЕДЕЛЕНИЕ ЗНАЧЕНИЙ ЛОГИЧЕСКИХ ВЫРАЖЕНИЙ

Выражения, которые принимают логические значения (истина или ложь) в результате выполнения операций сравнения (больше >, меньше <, больше или равно ≥, меньше или равно ≤, равно =, не равно ≠), также являются логическими выражениями. Кроме операций сравнения и логических операций такие выражения могут включать функции и алгебраические операции. Приоритет выполнения этих операций таков:

  1. Вычисление значений функций.
  2. Выполнение алгебраических операций (вначале возведение в степень, затем умножение и деление, после чего вычитание и сложение).
  3. Выполнение операций сравнения (в порядке записи).
  4. Выполнение логических операций (сначала операции отрицания, затем операции логического умножения, потом операции логического сложения, последними выполняются операции импликации и эквивалентности).

Если в логическом выражении используются скобки, то сначала выполняются заключенные в них операции.

Пример 5
Для какого из приведенных ниже значений числа М истинно следующее выражение?
¬М ≥ 10 ∧ M > 3

1) 1          2) 2         3) 3        4) 4

Решение. В соответствии с приоритетами операций сначала следует выполнить операции сравнения, затем отрицания, а потом — конъюнкцию. Отрицанием высказывания М ≥ 10 является высказывание М < 10. Получим выражение М < 10 ∧ M > 3. Для того чтобы это выражение (конъюнкция) было истинным, должны выполняться (т. е. быть истинными) оба неравенства. Следовательно, значение М должно быть больше 3, но меньше 10. Среди предложенных значений этому условию удовлетворяет только одно — число 4.
Ответ: 4) 4.

Задачи, подобные предыдущему примеру, можно решать и с помощью таблиц истинности.

Пример 6.
Для какого из приведенных ниже значений числа М истинно следующее выражение?
¬М ≥ 10 ∧ M > 3

1) 1           2) 2         3) 3        4) 4

Решение. Составим таблицу истинности: все операции выражения укажем в столбцах таблицы, все предложенные значения М укажем в ее строках. Рассчитаем значения таблицы:

Последний столбец содержит результат всего выражения. Истинным оно будет только для значения числа М, равного 4.
Ответ: 4) 4.

Пример 7.
В табличной форме представлены ежемесячные данные о продаже групп товаров за полгода. Сколько групп товаров демонстрировали рост продаж в весенние месяцы или вышли на уровень свыше 80 % в июне?

Решение. Переформулируем условие задачи: необходимо найти группы товаров, для которых (Март < Апрель) ∧ (Апрель < Май) v (Июнь > 80).

Введем обозначения:
А = (Март < Апрель)
В = (Апрель < Май)
С = (Июнь > 80)

Тогда выражение можно записать как А ∧ В v С.

Логическое выражение состоит из одной конъюнкции и одной дизъюнкции. Значение выражения конъюнкции истинно только тогда, когда истинны оба составляющие его простых выражения ((Март < Апрель) и (Апрель < Май)). Значение выражения дизъюнкции будет истинным, если хотя бы одно из составляющих его простых высказываний будет истинным.

Составим таблицу истинности для исходных данных.

Логическому выражению удовлетворяют 3 записи — 4–я, 6–я и 7–я.
Ответ: 3.


Конспект урока по информатике «Логические значения, операции, выражения».

Вернуться к Списку конспектов по информатике.

Математическая логика — это раздел математики, изучающий математические обозначения, формальные системы, доказуемость математических суждений, природу математического доказательства в целом, вычислимость и прочие аспекты оснований математики.

Алгебра высказываний

Под высказыванием понимаем всякое утверждение (повествовательное предложение), про которое всегда определенно и объективно можно сказать, является оно истинным или ложным. Например, «5-3 = 2» или «В неделе семь дней» — истинные высказывания, а «5 > 8» или «В русском языке 35 букв» — ложные высказывания. Синонимами слова «высказывания» можно считать: логическое высказывание, булевское выражение, суждение, утверждение и т.п. Фразы: «Ура!», «Который час?» — не являются высказываниями.

Если высказывание истинное, то ему предписывается значение «истина» (другие обозначения: «1», «ДА» , «И», «+», «true»). Ложному высказыванию предписывается значение «ложь» (другие обозначения: «О», «НЕТ», «Л», «-«, «false»). Совокупность возможных значений высказывания образует множество истинности {0,1} и {И,Л}.

Есть два вида высказываний: простые и составные (сложные). Под простым будем понимать высказывание, которое не может быть разбито на более простые высказывания. Про него всегда однозначно можно сказать, что оно истинно или ложно, не интересуясь его структурой. Из простых высказываний при помощи логических операций можно строить сложные высказывания, которые всегда только истинны или только ложные. Высказывания обозначаются заглавными латинскими буквами: Математическая логикасегодня вторникМатематическая логика если студент успешно сдал сессионные экзамены, то переводится на следующий курс и будет получать стипендию».

Логические операции

Операции над высказываниями задают в виде таблиц, называемых таблицами истинности.

Отрицание высказывания

Для каждого высказывания А может быть сформировано новое высказывание Математическая логикаотрицание высказывания А, которое истинно, когда А ложно, и ложно, когда А истинно. Символ Математическая логика соответствует логическому союзу «не». Математическая логикачитается «не А» или «не верно, что А». Отрицание — одноместная (или унарная) операция. Последующие операции — двухместные (или бинарные). Например, если Математическая логикаМатематическая логика истинное высказывание, то Математическая логикаложное высказывание (отрицание А), или если Математическая логикав комнате холодно», Математическая логикав комнате не холодно». Отметим, что высказывание «в комнате жарко» не является отрицанием В.

Математическая логика

Конъюнкция высказываний

Конъюнкцией высказываний А и В называется высказывание Математическая логика которое истинно только в том случае, когда и А, и В одновременно истинны. Выражение Математическая логика читается «А и В». Пример: пусть Математическая логикаделится на Математическая логикаделится на 4″. Тогда формула Математическая логика имеет смысл: «12 делится на 3 и на 4».

Операцию конъюнкции можно определить и для нескольких высказываний как связку высказываний, объединенных союзом «и». Конъюнкция из п высказываний — новое высказывание, причем высказывание

Математическая логика

имеет значение «истина», если Математическая логика истинны. Во всех других случаях эта конъюнкция имеет значение «ложь». Пусть, например, Математическая логикаотец старше сынаМатематическая логика Мурманск севернее Смоленска». Тогда высказывание Математическая логика («8=3 и отец старше сына, и

Мурманск севернее Смоленска») — ложное высказывание. В то время как Математическая логика и отец старше сына, и Мурманск севернее Смоленска» — истинное высказывание.

Дизъюнкция высказываний

Дизъюнкцией высказываний А и В называется высказывание Математическая логикакоторое ложно только тогда, когда и А, и В ложны одновременно. Дизъюнкция имеет значение «истина», если хотя бы одно из высказываний, входящее в дизъюнкцию, является истинным. Выражение Математическая логикачитается «А или В». Пусть Математическая логика Тогда Математическая логика

Операцию дизъюнкции можно определить для нескольких высказываний как связку высказываний, объединенных союзом «или»,

Математическая логика

В этом случае высказывание А истинно, если истинно хотя бы одно из высказываний, входящих в связку.

Импликация высказываний

Импликацией высказываний А и В называется высказывание Математическая логика которое ложно только в том случае, когда А — истинно, а В — ложно. Во всех других случаях импликация Математическая логика имеет значение «истина». Символ Математическая логика соответствует логическому союзу: «если А, то В». Например, А — «целое число делится на 4, то оно делится на 2». Для иллюстрации содержательного смысла импликации рассмотрим следующий пример: А Математическая логика«папа завтра получит премию», В Математическая логика «папа завтра купит сыну велосипед». Тогда импликация Математическая логика может быть сформулирована так: «если папа завтра получит премию, то купит сыну велосипед».

Математическая логика

Пусть А и В истинны. Тогда папа, получив премию, покупает сыну велосипед. Естественно считать это истинным высказыванием. Когда же папа не купит сыну велосипед (В — ложно), получив премию (А — истинно), то это, мягко говоря, не логичный поступок, а импликация имеет значение «ложь». Если же папа не получит премию (А — ложно), но купит велосипед (В -истинно), то результат положителен. В том случае, если, не получив премии (А ложно), папа не купит велосипед (В — ложно) -обещание не нарушено, результат можно считать истинным.

Эквивалентность высказываний

Эквивалентностью высказываний А и В называется высказывание Математическая логика которое истинно, когда высказывания и А, и В оба истинны или оба. ложны. Символ логической эквивалентности Математическая логикасоответствует связке «тогда и только тогда». Пример. Пусть А Математическая логика «число ЗМатематическая логика является четным», В Математическая логика «число Математическая логика является четным». Высказывание «число ЗМатематическая логика является четным тогда и только тогда, когда Математическая логика -четное число» есть эквивалентность высказываний А и В. Эквивалентность высказываний может быть задана следующей таблицей истинности:

Математическая логика

Замечание. Характерной особенностью операций над высказываниями является введение логических союзов с точно определенным смыслом, не допускающим никакой двусмысленности в толковании этих символов. Таким образом, математическая логика применима не для любых высказываний, а только для таких, которые допуск кают четкую оценку в двоичной системе «истина — ложь». Для преодоления такого рода ограничений в рамках нечеткой математики разрабатывается нечеткая логика.

Если в выражении встречаются различные логические операции, то в качестве естественного порядка (выполняемого поочередно слева направо) используется следующая последовательность: Математическая логика Это означает, что сначала выполняются операции отрицания, затем конъюнкции и т. д. Для нарушения порядка служат скобки. Рассмотрим пример. Пусть высказывания А и В имеют значения «истина», а высказывания С и Б — «ложь». Тогда формула Математическая логика имеет значение «ложь», т.к.:

Математическая логика

Введя скобки, получим формулу Математическая логика которая уже имеет другое значение — «истина». Действительно:

Математическая логика

Если в выражении присутствуют арифметические операции, операции сравнения и логические операции, то порядок старшинства операций следующий:

Использование различных операций позволяет в удобной аналитической форме задавать различные множества.

Например, множество точек А, заштрихованное на рис. 1.16, может быть задано следующей формулой:

Математическая логика

Математическая логика

Система операций Математическая логика называется полной, если всякая формула эквивалентна некоторой формуле, в которую входят только операции из системы Математическая логика. Система введенных пяти операций (отрицания, конъюнкции, дизъюнкции, импликации и эквивалентности) полная, хотя вообще говоря, избыточна, так как одни логические операции могут быть выражены через другие. Например, импликация и эквивалентность можно выразить через отрицание, конъюнкцию и дизъюнкцию следующим образом:

Математическая логика

Булевы функции

Всякую формулу логики высказываний можно рассматривать как некоторую функцию: каждая буква (высказывание) может принимать одно из двух значений — «истина» или «ложь», при этом сложное высказывание, заданное этой формулой, также может быть истинным или ложным. Так формула

Математическая логика

выражает функцию от переменных А, В и С.

Такого рода функции называются булевыми, а их аргументы — булевыми переменными. Аргументы булевых функций могут представлять собой, сокращенные обозначения некоторых конкретных высказываний. Тогда функция обозначает сокращенную запись некоторого сложного высказывания. Например, Математическая логика делится на 3», С ? «Мурманск севернее Смоленска». В этом случаеМатематическая логика «если Математическая логика делится на 3 и Мурманск севернее Смоленска». Сравните с известной формулой физики Математическая логика где m — масса тела, а — его ускорение, а F — сила, вызвавшая это ускорение. Буквы в булевых функциях могут выступать в качестве переменных. Подставляя вместо них любые высказывания, можно по формуле вычислить соответствующее значение функции. Действительно, если в формуле Математическая логикаМатематическая логика «истина», Y — «ложь», Z — «истина», то Математическая логика — «истина». Если же Z будет иметь ложное значение, то Математическая логика поменяет значение на противоположное и будет «ложью».

Целый ряд булевых функций обладает тем свойством, что они принимают одни и те же значения при любых значениях истинности аргументов. Такие формулы называются тождественно истинными. Например, при любых X и Y истинны формулы Математическая логика Тождественно ложные функции при любых значениях аргументов имеют значение «ложь». Так формулы Математическая логика всегда имеют значение «ложь».

Наиболее важные тождественно истинные формулы получили название Основные законы математической логики.

Основные законы математической логики

1.Коммутативность

Математическая логика

2.Ассоциативность

Математическая логика

3.Дистрибутивность

Математическая логика

4.Законы де Моргана

Математическая логика

5.Закон поглощения

Математическая логика

6.Закон идемпотентности

Математическая логика

Математическая логика

8.Закон противоречия

Математическая логика

9.Закон исключения третьего

Математическая логика

10.Закон двойного отрицания

Математическая логика

Пример:

Упростить выражение, используя тождественны преобразования

Математическая логика

Существует бесконечное множество тавтологий. Некоторы из них легли в основу методов доказательства.

Основные методы доказательств

При построении любой теории выделяется некоторый набор высказываний, так называемых аксиом, истинность которых постулируется. Из аксиом чисто логическим путем может был установлена истинность некоторых других высказываний называемых теоремами. Последовательность высказываний рассматриваемой теории, каждое из которых либо является аксиомой, либо выводится из одного или более предыдущих высказываний этой последовательности по логическим правилам вывода, называется доказательством. Высказывание, которое можно доказать, называется теоремой.

Формально каждая теорема может быть выражена в форме импликации Математическая логика где посылка А называется условием теоремы, а следствие В — заключением. Теорема верна, если выражающая ее импликация тождественно истинна, т. е. является тавтологией. Тавтологии рассматривают как некоторые логически истинные схемы рассуждений. В этой связи тавтологии играют роль законов, определяющих построение правильных умозаключений. Существует бесконечное множество тавтологий. Некоторые из них легли в основу методов доказательства. Основные методы доказательств.

Метод цепочек импликаций

Метод цепочек импликаций состоит в том, что из посылки А страивается цепочка из Математическая логика-импликаций, последним высказыванием в которой является заключение теоремы В, т. е.

Математическая логика

В основе этого метода лежит закон цепного высказывания или закон силлогизма Математическая логика

Метод от противного

Метод от противного. Используя этот метод, вместо доказательства прямого следствия «из А следует В» доказывают, что из «не В» следует «не А». Этот метод основан на законе контрапозиций, имеющем следующий вид:

Математическая логика

Метод необходимого и достаточного

Метод необходимого и достаточного. Теорема формулируется так: «Чтобы имело место А, необходимо и достаточно выполнение В». Доказательство такого вида теоремы распадается на две части:

а) доказывается, что если имеет место А, то справедливо В (В необходимо для А);

б) если имеет место В, то имеет место и А (В достаточно для А).

Доказательство таким методом базируется на законе тавтологии:

Математическая логика

Алгебра предикатов

Предикатом Математическая логика заданным на множествах Математическая логикаМатематическая логика

называется функция Р, отображающая их прямое произведение на двоичное множество, т. е. Математическая логикаМатематическая логика Множество М называется предметной областью предиката, Математическая логика называются предметными переменными или термами. Предикат представляет собой логическую функцию, принимающую, как и булевская функция, значение «истина» или «ложь», когда ее предметные переменные принимают определенные значения.

Рассмотрим примеры, Математическая логика одноместный предикат на множестве комплексных чисел, при этом, например, если Математическая логика истинное высказывание, а

положив Математическая логика получим Математическая логика «ложь». Выражение «X — брат Y» — двухместный предикат, заданный на множестве людей. Здесь термы X и Y указывают места, на которые нужна поставить имена двух людей, чтобы получить правильно построенное высказывание. Очевидно, что X — лицо мужского пола, а Y может выбираться из всего множества людей.

Всякий предикат Математическая логика определяет отношение R, такое, что Математическая логика тогда и только тогда, когда Математическая логикаМатематическая логика«истина». В этом случае говорят, что отношение R задается областью истинности предиката Математическая логика. Например, отношение Математическая логика «расстояние на плоскости между точками Математическая логика больше величины 1″ можно задать предикатом Математическая логика

Если в Математическая логика-местный предикат на место одного из термов подставить определенный элемент из соответствующего множества, то предикат станет Математическая логикаместным. Заменив все термы на конкретные значения из предметной области предиката, получим 0 — местный предикат, т. е. высказывание. Например, «Х- брат Y» — двухместный предикат, «X — брат Маши» — одноместный предикат, «Саша — брат Маши» — высказывание.

Логические операции над предикатами

Отрицание предиката

Пусть предикат Математическая логика задан на множествах Математическая логика Предикат Математическая логика называется отрицанием предиката Математическая логика тогда и только тогда, если при одних и тех же кортежах Математическая логика Математическая логика высказывание Математическая логика истинно, когда Математическая логика ложно и наоборот. Обозначение

Математическая логика

Например, предикат «Математическая логика— четное число» есть отрицание предиката «Математическая логика— нечетное число» на множестве целых чисел.

Конъюнкция предикатов

Пусть на множествах Математическая логика заданы два Математическая логика— местных предиката Математическая логика и Математическая логика. Конъюнкцией этих предикатов называется предикат

Математическая логика

который истинен для одних и тех же кортежей только тогда, когда оба предиката — и Математическая логика и Математическая логика истинны.

Например, конъюнкция предикатов Математическая логика где Математическая логика вещественные числа, определяет предикат «точки правой половины единичного круга» (см. рис. 1.17а).

Дизъюнкция предикатов

Дизъюнкция предикатов Математическая логика и Математическая логика есть новый предикат Математическая логикакоторый имеет значение «ложь» для тех и только тех кортежей из Математическая логика для которых оба предиката — и Математическая логика и Математическая логика — имеют значение «ложь». На рис. 1.17 6 иллюстрируется дизъюнкция предиката Математическая логика(заштрихованная область).

Импликация предикатов

Импликация предикатов Математическая логика и Математическая логика есть новый предикат Математическая логика который имеет значение «ложь» для тех и только тех кортежей из Математическая логика для которых предикат Математическая логика имеет значение «истина», а предикат Математическая логика имеет значение «ложь».

Например, импликация «Математическая логика делится на 4″ —» » Математическая логика делится на 2″ есть предикат: «если Математическая логика делится на 4, то Математическая логика делится на 2″.

Математическая логика

Эквивалентность предикатов

Эквивалентность предикатов Математическая логика и Математическая логика есть новый предикат Математическая логикаМатематическая логика который имеет значение «истина» для тех и только тех кортежей из Математическая логика для которых предикат Математическая логика и предикат Математическая логика имеют одинаковые значение или оба «истина» или оба «ложь». Два предиката, заданные на одних и тех же множествах, называются равносильными, если при всех наборах входящих в них предметных переменных эти предикаты принимают одинаковые значения. Равносильность называют также логической эквивалентностью. Например, эквивалентность предикатов Математическая логика делится на 6» и Математическая логикаделится на 2 и Математическая логика делится на 3» есть предикат Математическая логикаМатематическая логика «если Математическая логика делится на 6, то Математическая логика делится на 2 и на 3». Предикаты Математическая логика логически эквивалентны.

Наряду с логическими операциями важную роль играют операции, называемые кванторами. Квантор всеобщности есть операция, которая предикат Математическая логика превращает в высказывание: «все Математическая логика обладают свойством Математическая логика». Знак квантора всеобщности Математическая логика Он заменяет фразы: «для всех», «каждый», «любой» и т.п. Обозначение Математическая логика читается так: «для всех Математическая логика таких, что Р от Математическая логика». Например, Математическая логика вещественное число», есть предикат «Математическая логика — положительное число». Тогда Математическая логика есть высказывание «каждое число — положительно». Это ложное высказывание. Если же Математическая логика — любое натуральное число Математическая логика то Математическая логика есть выражение: «каждое натуральное число — положительно» — истинное высказывание. Квантор всеобщности есть обобщение серии конъюнкций единичных высказываний. Пусть М — множество очков, которое может выпасть при бросании игральной кости, т. е. Математическая логика предикат: «при бросании игральной кости один раз выпадает Математическая логика очков», где Математическая логика. Применение квантора всеобщности позволяет вместо сложного высказывания Математическая логиказаписать равносильное ему компактное высказывание Математическая логикаМатематическая логика: «при бросании игральной кости один раз может выпасть любое из шести первых натуральных чисел».

Квантор существования

Квантор существования есть операция, которая предикат Математическая логика превращает в высказывание: «существует хотя бы один Математическая логика

из М, обладающий свойством Математическая логика». Знак квантора существования Математическая логика Он заменяет фразы: «существует хотя бы один», «найдется», «некоторый» и т.п. Обозначение Математическая логика читается так: «существует хотя бы один Математическая логика такой, что Р от Математическая логика». Например, Математическая логика — предикат: «Математическая логика — студент», где Математическая логика — элемент множества жителей Москвы. Тогда выражение Математическая логика есть высказывание «хотя бы один житель Москвы является студентом». Квантор существования есть обобщение серии дизъюнкций единичных высказываний. Если задано множество Математическая логика и на нем определен предикат Математическая логика

Кванторы обладают свойствами, являющимися аналогами законов де Моргана:

Математическая логика

С помощью кванторов можно выражать ряд часто используемых на практике отношений между множествами. Например, высказывание «все объекты Математическая логика из данного множества, обладающие свойством Математическая логика, обладают также и свойством Математическая логика» формально можно записать — Математическая логика

Переход от Математическая логика или Математическая логика называется квантификацией или связыванием переменной Математическая логика. Связанная переменная фактически не является переменной, т. е. переход от Математическая логикаМатематическая логика или от Математическая логика не меняет истинности выражений. Навешивание переменной на многоместный предикат уменьшает в нем число свободных переменных и превращает его в предикат от меньшего числа переменных

Рассмотрим пример. На множестве чисел задан двухместный предикат Математическая логика число Математическая логика делится на число Математическая логика». Связывая одну переменную, можно получить следующие одноместные предикаты:

Математическая логика«каждое число делится на Математическая логика» — ложь;

Математическая логика«существует число, которое делится на Математическая логика» — истина;

Математическая логика«число Математическая логика делится на любое число» — ложь;

Математическая логика«существует число, на которое делится Математическая логика» — истина.

Связывая обе переменные данного предиката, получим высказывания:

Математическая логика«каждое число делится на любое число» -ложное высказывание,

Математическая логика«существует число, на которое делится любое число» — истина, т.к. такое число есть 1,

Математическая логика«существует число, которое делится на любое число» — ложное высказывание,

Математическая логика«существует число, которое делится на какое-нибудь число» — истинное высказывание.

Решение заданий и задач по предметам:

  • Математика
  • Высшая математика
  • Математический анализ
  • Линейная алгебра

Дополнительные лекции по высшей математике:

  1. Тождественные преобразования алгебраических выражений
  2. Функции и графики
  3. Преобразования графиков функций
  4. Квадратная функция и её графики
  5. Алгебраические неравенства
  6. Неравенства
  7. Неравенства с переменными
  8. Прогрессии в математике
  9. Арифметическая прогрессия
  10. Геометрическая прогрессия
  11. Показатели в математике
  12. Логарифмы в математике
  13. Исследование уравнений
  14. Уравнения высших степеней
  15. Уравнения высших степеней с одним неизвестным
  16. Комплексные числа
  17. Непрерывная дробь (цепная дробь)
  18. Алгебраические уравнения
  19. Неопределенные уравнения
  20. Соединения
  21. Бином Ньютона
  22. Число е
  23. Непрерывные дроби
  24. Функция
  25. Исследование функций
  26. Предел
  27. Интеграл
  28. Двойной интеграл
  29. Тройной интеграл
  30. Интегрирование
  31. Неопределённый интеграл
  32. Определенный интеграл
  33. Криволинейные интегралы
  34. Поверхностные интегралы
  35. Несобственные интегралы
  36. Кратные интегралы
  37. Интегралы, зависящие от параметра
  38. Квадратный трехчлен
  39. Производная
  40. Применение производной к исследованию функций
  41. Приложения производной
  42. Дифференциал функции
  43. Дифференцирование в математике
  44. Формулы и правила дифференцирования
  45. Дифференциальное исчисление
  46. Дифференциальные уравнения
  47. Дифференциальные уравнения первого порядка
  48. Дифференциальные уравнения высших порядков
  49. Дифференциальные уравнения в частных производных
  50. Тригонометрические функции
  51. Тригонометрические уравнения и неравенства
  52. Показательная функция
  53. Показательные уравнения
  54. Обобщенная степень
  55. Взаимно обратные функции
  56. Логарифмическая функция
  57. Уравнения и неравенства
  58. Положительные и отрицательные числа
  59. Алгебраические выражения
  60. Иррациональные алгебраические выражения
  61. Преобразование алгебраических выражений
  62. Преобразование дробных алгебраических выражений
  63. Разложение многочленов на множители
  64. Многочлены от одного переменного
  65. Алгебраические дроби
  66. Пропорции
  67. Уравнения
  68. Системы уравнений
  69. Системы уравнений высших степеней
  70. Системы алгебраических уравнений
  71. Системы линейных уравнений
  72. Системы дифференциальных уравнений
  73. Арифметический квадратный корень
  74. Квадратные и кубические корни
  75. Извлечение квадратного корня
  76. Рациональные числа
  77. Иррациональные числа
  78. Арифметический корень
  79. Квадратные уравнения
  80. Иррациональные уравнения
  81. Последовательность
  82. Ряды сходящиеся и расходящиеся
  83. Тригонометрические функции произвольного угла
  84. Тригонометрические формулы
  85. Обратные тригонометрические функции
  86. Теорема Безу
  87. Математическая индукция
  88. Показатель степени
  89. Показательные функции и логарифмы
  90. Множество
  91. Множество действительных чисел
  92. Числовые множества
  93. Преобразование рациональных выражений
  94. Преобразование иррациональных выражений
  95. Геометрия
  96. Действительные числа
  97. Степени и корни
  98. Степень с рациональным показателем
  99. Тригонометрические функции угла
  100. Тригонометрические функции числового аргумента
  101. Тригонометрические выражения и их преобразования
  102. Преобразование тригонометрических выражений
  103. Комбинаторика
  104. Вычислительная математика
  105. Прямая линия на плоскости и ее уравнения
  106. Прямая и плоскость
  107. Линии и уравнения
  108. Прямая линия
  109. Уравнения прямой и плоскости в пространстве
  110. Кривые второго порядка
  111. Кривые и поверхности второго порядка
  112. Числовые ряды
  113. Степенные ряды
  114. Ряды Фурье
  115. Преобразование Фурье
  116. Функциональные ряды
  117. Функции многих переменных
  118. Метод координат
  119. Гармонический анализ
  120. Вещественные числа
  121. Предел последовательности
  122. Аналитическая геометрия
  123. Аналитическая геометрия на плоскости
  124. Аналитическая геометрия в пространстве
  125. Функции одной переменной
  126. Высшая алгебра
  127. Векторная алгебра
  128. Векторный анализ
  129. Векторы
  130. Скалярное произведение векторов
  131. Векторное произведение векторов
  132. Смешанное произведение векторов
  133. Операции над векторами
  134. Непрерывность функций
  135. Предел и непрерывность функций нескольких переменных
  136. Предел и непрерывность функции одной переменной
  137. Производные и дифференциалы функции одной переменной
  138. Частные производные и дифференцируемость функций нескольких переменных
  139. Дифференциальное исчисление функции одной переменной
  140. Матрицы
  141. Линейные и евклидовы пространства
  142. Линейные отображения
  143. Дифференциальные теоремы о среднем
  144. Теория устойчивости дифференциальных уравнений
  145. Функции комплексного переменного
  146. Преобразование Лапласа
  147. Теории поля
  148. Операционное исчисление
  149. Системы координат
  150. Рациональная функция
  151. Интегральное исчисление
  152. Интегральное исчисление функций одной переменной
  153. Дифференциальное исчисление функций нескольких переменных
  154. Отношение в математике
  155. Графы в математике
  156. Линейные пространства
  157. Первообразная и неопределенный интеграл
  158. Линейная функция
  159. Выпуклые множества точек
  160. Система координат

Высказывания. Операции над высказываниями

Определение высказываний

Высказывание — утверждение, относительно которого можно сказать истинно (1, истина, true) оно или ложно (0, ложь, false).

Высказывания обозначаются заглавными латинскими буквами %%A, B, C, …%% или буквами с индексами %%A_1, B^2, C’, …%%.

Примеры

Следующие предложения являются высказываниями:

%%A_1%%: «Лондон — столица Австрии».
%%A_2%%: «Число 8 больше числа 3».
%%A_3%%: «Число 8 больше числа 13».
%%A_4%%: «Луна — спутник планеты Земля».

Причем высказывания %%A_1, A_3%% — ложные, а %%A_2, A_4%% — истинные.

Следующие предложения не являются высказываниями:

%%B_1%%: «Какой сегодня день недели?».
%%B_2%%: «%%2 + 3%%».
%%B_3%%: «Число %%x%% больше 3».

Мы не можем сказать о любом из высказываний %%B_1, B_2, B_3%% истинно оно или ложно. Например, в предложении %%B_3%% буква %%x%% — переменная. Если поставить какое либо значение вместо нее, например 8, то получим истинное высказывание.

Операции над высказываниями

Сложные высказывания построены из более простых, используя следующие логические знаки
$$
land, lor, rightarrow, leftrightarrow, overline{},
$$
которые имеют соответствующие названия: конъюнкция (логическое И), дизъюнкция (логическое ИЛИ), импликация (логические следование), эквиваленция (логическое равенство) и отрицание (логическое НЕ).

Пусть %%A%% и %%B%% — некоторые высказывания.

Конъюнкция

Конъюнкцией высказываний %%A%% и %%B%%

называется новое высказывание, обозначаемое %%A land B%%, которое является истинным тогда и только тогда, когда высказывания %%A%% и %%B%% истины. Читается как %%A%% и %%B%%.

Рассмотрим конъюнкцию высказывний %%A_1%% и %%A_2%%, которая записывается как %%A_1 land A_2%% и читается как «Генуя — столица Австрии и число 8 больше числа 3». Это высказывание ложно, так как высказывание %%A_1%% ложно. Другими словами, конъюнкция является ложной тогда и только тогда, когда хотя бы одно из высказываний ложно.

Рассмотрим произвольные высказывания %%A%% и %%B%% и полученное из них высказывание %%A land B%%. Высказывания %%A, B%% могут быть как ложными, так и истинными. Возможны следующие варианты:

  1. %%A%% ложно, %%B%% ложно;
  2. %%A%% ложно, %%B%% истинно;
  3. %%A%% истинно, %%B%% ложно;
  4. %%A%% истинно, %%B%% истинно;

В каждом их этих случаев, вычислив значение конъюнкции высказываний %%A land B%%, получим следующую таблицу, которая называется таблицей истинности.

%%A%% %%B%% %%A land B%%
%%0%% %%0%% %%0%%
%%0%% %%1%% %%0%%
%%1%% %%0%% %%0%%
%%1%% %%1%% %%1%%

Где %%1%% обозначает истинное высказывание, %%0%% — ложное высказывание.

Операцию конъюникции можно распространить и на несколько высказываний. Пусть %%A_1, A_2, …, A_n%% — высказывания. Тогда высказывание %%A_1 land A_2 land … land A_n%%, являющееся конъюнкцией высказываний %%A_1, A_2, …, A_n%%, будет истинным тогда и только тогда, когда все высказывания будут истинными.

Дизъюнкция

Дизъюнкцией высказываний %%A%% и %%B%%

называется новое высказывание, обозначаемое %%A lor B%%, которое является ложным тогда и только тогда, когда высказывания %%A%% и %%B%% ложны.
Читается как %%A%% или %%B%%.

Рассмотрим дизъюнкцию высказывний %%A_1%% и %%A_2%%, которая записывается как %%A_1 lor A_2%% и читается как «Москва — столица Австрии или число 8 больше числа 3». Это высказывание истинно, так как высказывание %%A_2%% истинно. Другими словами, дизъюнкция является истинной тогда и только тогда, когда хотя бы одно из высказываний истино.

Таблица истинности для дизъюнкции выглядит следующим образом.

%%A%% %%B%% %%A lor B%%
%%0%% %%0%% %%0%%
%%0%% %%1%% %%1%%
%%1%% %%0%% %%1%%
%%1%% %%1%% %%1%%

Аналогично конъюнкции, операцию дизъюнкции можно распространить и на несколько высказываний. Пусть %%A_1, A_2, …, A_n%% — высказывания. Тогда высказывание %%A_1 lor A_2 lor … lor A_n%%, являющееся дизъюнкцией высказываний %%A_1, A_2, …, A_n%%, будет ложным тогда и только тогда, когда все высказывания будут ложными.

Импликация

Импликацией высказываний %%A%% и %%B%% называется

новое высказывание, обозначаемое %%A rightarrow B%%, которое является ложным тогда и только тогда, когда высказывание %%A%% истинно, %%B%% ложно. Читается как: «Если %%A%%, то %%B%%»; «%%A%% влечет %%B%%»; «из %%A%% следует %%B%%»; «%%A%% достаточно для %%B%%»; %%B%% необходимо для %%A%%».

Рассмотрим импликацию высказывний %%A_2%% и %%A_1%%, которая записывается как %%A_2 rightarrow A_1%% и читается как «Если число %%8%% больше числа %%3%%, то Москва — столица Австрии». Это высказывание ложно, так как высказывание %%A_2%% истинно, а %%A_1%% ложно.

Таблица истинности для импликации выглядит следующим образом.

%%A%% %%B%% %%A rightarrow B%%
%%0%% %%0%% %%1%%
%%0%% %%1%% %%1%%
%%1%% %%0%% %%0%%
%%1%% %%1%% %%1%%

Эквиваленция

Эквиваленцией высказываний %%A%% и %%B%%

называется новое высказывание, обозначаемое %%A leftrightarrow B%%, которое является истинным тогда и только тогда, когда высказывание %%A%% и %%B%% одновременно истинны или ложны.
Читается как: «%%A%% равносильно %%B%%»; «%%A%% необходимо и достаточно для %%B%%»; «%%A%% тогда и только тогда, когда %%B%%».

Рассмотрим импликацию высказывний %%A_1%% и %%A_2%%, которая записывается как %%A_1 leftrightarrow A_2%% и читается как «Москва — столица Австрии тогда и только тогда, когда число %%8%% больше числа %%3%%». Это высказывание ложно, так как высказывание %%A_2%% истинно, а %%A_1%% ложно.

Таблица истинности для эквиваленции выглядит следующим образом.

%%A%% %%B%% %%A leftrightarrow B%%
%%0%% %%0%% %%1%%
%%0%% %%1%% %%0%%
%%1%% %%0%% %%0%%
%%1%% %%1%% %%1%%

Также эквиваленцию можно выразить через импликацию и конъюнкцию, тогда

$$
A leftrightarrow B = (A rightarrow B) land (B rightarrow A)
$$

Покажем это, используя таблицы истинности.

%%A%% %%B%% %%A leftrightarrow B%% %%A rightarrow B%% %%B rightarrow A%% %%(A rightarrow B) land (B rightarrow A)%%
%%0%% %%0%% %%1%% %%1%% %%1%% %%1%%
%%0%% %%1%% %%0%% %%1%% %%0%% %%0%%
%%1%% %%0%% %%0%% %%0%% %%1%% %%0%%
%%1%% %%1%% %%1%% %%1%% %%1%% %%1%%

Как видно из таблицы истинности столбцы %%A leftrightarrow B%% и %%(A rightarrow B) land (B rightarrow A)%% имеют одни и те же значения при одинаковых наборах значений %%A%% и %%B%%, что говорит о равенстве этих двух формул.

Отрицание

Отрицанием высказывания %%A%%

называется новое высказывание, обозначаемое %%overline{A}%%, которое является истинным, когда высказывание %%A%% ложно, и ложным, когда высказываине %%A%% истинно.
Читается как: «не %%A%%»; «неверно, что %%A%%».

Рассмотрим отрицание высказывния %%A_1%%, которое записывается как %%overline{A_1}%% и читается как «неверно, что Москва — столица Австрии». Это высказывание истинно, так как высказывание %%A_1%% ложно.

Таблица истинности для отрицания выглядит следующим образом.

%%A%% %%overline{A}%%
%%0%% %%1%%
%%1%% %%0%%

Алгебра высказываний и операции над высказываниями

Высказывание — первый важнейший объект изучения математической логики. Алгебра высказываний изучает способы построения высказываний из уже имеющихся высказываний, закономерности таких способов сочетания высказываний. Алгебра высказываний является фундаментом математической логики.

Понятие высказывания

Предметом исследования алгебры высказываний являются высказывания. Но алгебра высказываний не ставит целью их всестороннее изучение. Из многочисленных свойств высказывания алгебру высказываний интересует лишь одно: истинно оно или ложно. Именно это и является определяющим свойством высказывания. Итак, под высказыванием понимается такое предложение, которое либо истинно, либо ложно. Высказывание не может быть одновременно и истинным, и ложным.

В дальнейшем будем считать, что имеется первоначальная совокупность некоторых простейших высказываний, называемых элементарными или исходными, о каждом из которых точно известно, истинно оно или ложно. Причем в этой совокупности имеются как истинные высказывания, так и ложные.

Договоримся обозначать конкретные высказывания начальными заглавными буквами латинского алфавита A,B,C,D,ldots или теми же буквами с индексами внизу.

Приведем примеры высказываний, которые будут использованы в дальнейшем:

A_1colon «Москва — столица России»;
A_2colon «Саратов находится на берегу Невы»;
A_3colon «Все люди смертны»;
A_4colon «Сократ — человек»;
A_5colon «7 < 4»;
A_6colon «Волга впадает в Каспийское море»;
A_7colon «А.С.Пушкин — великий русский математик»;
A_8colon «Снег белый».

Обозначив истинное высказывание символом 1, а ложное — 0, введем функцию lambda, заданную на совокупности всех высказываний и принимающую значения в двухэлементном множестве {0;1}, по следующему правилу:

lambda(P)= begin{cases}1,&text{if}~~P~~text{is~true},\ 0,&text{if}~~P~~text{is~false}.end{cases}

Функция lambda называется функцией истинности, а значение lambda(P) — логическим значением или значением истинности высказывания P. Для приведенных высказываний имеем логические значения:

lambda(A_1)=1,~~ lambda(A_2)=0,~~ lambda(A_3)=1,~~ lambda(A_4)=1,~~ lambda(A_5)=0,~~ lambda(A_6)=1,~~ lambda(A_7)=0,~~ lambda(A_8)=1.

Отметим, что в литературе имеются следующие обозначения для истинных высказываний: 1, И, t (от англ. true — истинный) и для ложных высказываний: 0, Л, f (от англ. false — ложный). Из этих обозначений будем использовать 1 и 0. Это обусловлено рядом причин. Во-первых, таблицы истинности для формул алгебры высказываний принимают более лаконичный и стандартизированный вид, так как в этом случае наборы значений пропозициональных переменных можно расположить в порядке возрастания чисел, которые этими наборами закодированы в двоичной системе счисления. Например, для случая трех пропозициональных переменных X,,T,,Z набор значений этих переменных 000 означает двоичную запись десятичного числа 0, набор 001 — двоичную запись десятичного числа 1, набор 010 — двоичную запись десятичного числа 2, 011 — 3, 100 — 4, 101 — 5, 110 —6, 111 — 7. Во-вторых, более удобный и математически строгий вид принимают многие формулы и алгоритмы алгебры высказываний. В-третьих, обозначение 0 и 1 принято и более целесообразно в приложениях математической логики к компьютерам и информатике.

Из элементарных высказываний с помощью операций над высказываниями или логических связок строят сложные высказывания. Перейдем к точному описанию таких построений.


Отрицание высказывания

Определение 1.1. Отрицанием высказывания P называется новое высказывание, обозначаемое lnot P (читается: «не P» или «не верно, что P«), которое истинно, если высказывание P ложно, и ложно, если высказывание P истинно. Другими словами, логическое значение высказывания lnot P связано с логическим значением высказывания P, как указано в следующей таблице, называемой таблицей истинности операции отрицания:

begin{array}{|c||c|}hline lambda(P)& lambda(lnot P)\hline 0&1\hline 1&0\hline end{array}

Здесь может возникнуть вопрос, почему приписывание истинности или ложности высказыванию lnot P осуществляется именно на основании приведенной таблицы. Конечно, можно ответить, что об определениях не спорят. Но ведь мы желаем построить математическую теорию (алгебру высказываний), которая в какой-то мере отражала бы реально существующий в природе человеческого мышления процесс построения составных высказываний из элементарных и имела бы реальный смысл. Затем мы должны будем развить нашу математическую теорию, а полученные выводы применить в практике мышления и при этом не войти в противоречие с общеизвестными законами мышления. Определение отрицания с помощью приведенной таблицы (как, впрочем, и других логических связок с помощью соответствующих таблиц, о чем речь пойдет далее) появилось как результат длительного опыта, и оно полностью оправдало себя на практике.


Пример 1.2. Применим операцию отрицания к высказыванию A_6colon «Волга впадает в Каспийское море». Данное отрицание можно читать так: «Неверно, что A_6» т.е. «Неверно, что Волга впадает в Каспийское море». Или же частицу «не» переносят на такое место (чаще всего ставят перед сказуемым), чтобы получилось правильно составленное предложение: «Волга не впадает в Каспийское море». Таблица из определения 1.1 дает для данного высказывания следующее логическое значение: lambda(lnot A_6)= lnot lambda(A_6)= lnot1=0, т.е. высказывание lnot A_6 ложно. Ложность высказывания lnot A_6 обусловлена только истинностью исходного высказывания A_6 и определением 1.1, но никак не соображениями смысла (содержания) высказывания lnot A_6. Другое дело, что само определение 1.1 потому и имеет такую формулировку, что оно правильно (или, как говорят, адекватно) отражает факты, известные нам из практики.


Конъюнкция двух высказываний

Определение 1.3. Конъюнкцией двух высказываний P и Q называется новое высказывание, обозначаемое Pland Q или PAnd Q (читается: «P и Q«), которое истинно лишь в единственном случае, когда истинны оба исходных высказывания P и Q, и ложно во всех остальных случаях. Другими словами, логическое значение высказывания Pland Q связано с логическими значениями высказываний P и Q, как указано в следующей таблице, называемой таблицей истинности операции конъюнкции:

begin{array}{|c|c||c|}hline lambda(P)& lambda(Q)& lambda(Pland Q)\hline 0&0&0\hline 0&1&0\hline 1&0&0\hline 1&1&1\hline end{array}

Практика полностью подтвердила, что именно такое распределение значений истинности наиболее соответствует тому смыслу, который придается в процессе мыслительной деятельности связующему союзу «и».

Пример 1.4. Применим операцию конъюнкции к высказываниям A_2 и A_3. Получим высказывание A_2land A_3 л Л3: «Саратов находится на берегу Невы, и все люди смертны». Конечно, мы не воспринимаем это высказывание как истинное из-за первой, ложной, его части. К выводу о ложности полученного высказывания также придем, исходя из логических значений исходных высказываний A_2 и A_3 и определения 1.3 конъюнкции на основании приведенной там таблицы. В самом деле,

lambda(A_2land A_3)= lambda(A_2)land lambda(A_3)= 0land1=0.


Дизъюнкция двух высказываний

Определение 1.5. Дизъюнкцией двух высказываний P и Q называется новое высказывание, обозначаемое Plor Q (читается «P или Q«), которое истинно в тех случаях, когда хотя бы одно из высказываний P или Q истинно, и ложно в единственном случае, когда оба высказывания P и Q ложны. Другими словами, Plor Q — такое высказывание, логическое значение которого связано с логическими значениями исходных высказываний P и Q так, как указано в следующей таблице, называемой таблицей истинности операции дизъюнкции:

begin{array}{|c|c||c|}hline lambda(P)& lambda(Q)& lambda(Plor Q)\hline 0&0&0\hline 0&1&1\hline 1&0&1\hline 1&1&1\hline end{array}

Пример 1.6. Применим операцию дизъюнкцию к высказываниям A_3 и A_5. Получим составное высказывание A_3lor A_5colon «Все люди смертны, или 7&lt;4«. Несмотря на первоначально кажущуюся странность этого высказывания, нет сомнений в его истинности. К аналогичному заключению приводит также формальное вычисление логического значения данного высказывания по таблице из определения 1.5, исходя из логических значений высказываний A_3 и A_5:

lambda(A_3lor A_5)= lambda(A_3)lor lambda(A_5)= 1lor0=1.

В то же время высказывание «Саратов находится на берегу Невы, или А. С. Пушкин — великий русский математик», являющееся дизъюнкцией высказываний A_2 и A_7, безусловно, ложно, что полностью согласуется с формальным вычислением его логического значения по таблице из определения 1.5:

lambda(A_2lor A_7)= lambda(A_2)lor lambda(A_7)= 0lor0=0.


Импликация двух высказываний

Определение 1.7. Импликацией двух высказываний P и Q называется новое высказывание, обозначаемое Pto Q (читается: «если P, то Q«, или «из P следует Q«, или «P влечет Q«, или «P достаточно для Q«, или «Q необходимо для P«), которое ложно в единственном случае, когда высказывание P истинно, а Q — ложно, а во всех остальных случаях — истинно. Другими словами, логическое значение высказывания Pto Q связано с логическими значениями высказываний P и Q, как указано в следующей таблице, называемой таблицей истинности операции импликации:

begin{array}{|c|c||c|}hline lambda(P)& lambda(Q)& lambda()\hline 0&0&1\hline 0&1&1\hline 1&0&0\hline 1&1&1\hline end{array}

В высказывании Pto Q высказывание P называется посылкой или антецедентом, а высказывание Q — следствием или консеквентом.

При определении импликации с еще большей силой встает вопрос, почему именно такое распределение принято в ее таблице истинности. Последние две строки в ней достаточно хорошо согласуются с нашим пониманием выражения «если…, то…«. Их обоснованием могут служить следующие соображения. Импликация призвана отразить процесс рассуждения, умозаключения. Общая характеристика этого процесса следующая. Если мы исходим из истинной посылки и правильно (верно) рассуждаем, то мы приходим к истинному заключению (следствию, выводу). Другими словами, если мы исходили из истинной посылки и пришли к ложному выводу, значит, мы неверно рассуждали. В импликации Pto Q имеется посылка P, следствие Q и процесс рассуждения to. Процесс рассуждения как раз и моделируется результатом операции Pto Q. Приведенное соображение служит обоснованием результата 1to0=0, а также результата 1to1=1.

Определенные сомнения возникают при оценке адекватности первых двух строк в таблице, определяющей импликацию. В первой строке при ложной посылке и ложном следствии импликация признается истинной. Следующие два примера добавляют аргументы в пользу такого определения логического значения импликации в этом случае. Рассмотрим такое высказывание: «Если число делится на 5, то и его квадрат делится на 5». Его истинность не вызывает сомнения. В частности, мы могли бы сказать: «Если 10 делится на 5, то 10^2 делится на 5″ или «Если 11 делится на 5, то и 11^5 делится на 5″. В первом из этих высказываний и посылка, и следствие истинны, во втором — и посылка, и следствие ложны. Тем не менее оба этих высказывания истинны. Для большей убедительности второе высказывание можно сформулировать в сослагательной форме: «Если бы 11 делилось на 5, то и 11^2 делилось бы на 5″. Есть утверждения такого типа и в житейской речи, которые признаются вполне нормальными. Например, «Если ты можешь переплыть Черное море, то я — турецкий султан».

В пользу второй строки таблицы, когда импликация остается истинной при ложной посылке и истинном следствии, говорит такой пример. Высказывание «Если первое слагаемое делится на 5 и второе слагаемое делится на 5, то и сумма делится на 5», несомненно, истинно. Но, в частности, мы могли бы сказать: «Если 10 делится на 5 и 20 делится на 5, то 30 делится на 5» или «Если 12 делится на 5 и 13 делится на 5, то 25 делится на 5». В первом из этих высказываний и посылка истинна (как конъюнкция двух истинных выражений), и следствие истинно. Во втором же высказывании посылка ложна (как конъюнкция двух ложных высказываний), а следствие истинно. Тем не менее, как мы уже отметили, оба этих высказывания признаются истинными.

Пример 1.8. Высказывание A_6to A_5: «Если Волга впадает в Каспийское море, то 7&lt;4» ложно, так как

lambda(A_6to A_5)= lambda(A_6)to lambda(A_5)= 1to0=0.

Высказывание «Если Саратов находится на берегу Невы, то А. С. Пушкин — великий русский математик», являющееся импликацией высказываний A_2 и A_7, истинно, так как

lambda(A_2to A_7)= lambda(A_2)to lambda(A_7)= 0to0=1.


Эквивалентность двух высказываний

Определение 1.9. Эквивалентностью двух высказываний P и Q называется новое высказывание, обозначаемое P leftrightarrow Q (читается: «P эквивалентно Q«, или «P необходимо и достаточно для Q«, или «P тогда и только тогда, когда Q«, или «P, если и только если Q«), которое истинно в том и только в том случае, когда одновременно оба высказывания P и Q либо истинны, либо ложны, а во всех остальных случаях — ложно. Другими словами, логическое значение высказывания P leftrightarrow Q связано с логическими значениями высказываний P и Q, как указано в следующей таблице, называемой таблицей истинности операции эквивалентности:

begin{array}{|c|c||c|}hline lambda(P)& lambda(Q)& lambda(Pleftrightarrow Q)\hline 0&0&1\hline 0&1&0\hline 1&0&0\hline 1&1&1\hline end{array}

Пример 1.10. Высказывание «7&lt;4 тогда и только тогда, когда снег белый», являющееся эквивалентностью высказываний A_5 и A_8, ложно, так как

lambda(A_5leftrightarrow A_8)= lambda(A_5) leftrightarrow lambda(A_8)= 0leftrightarrow 1=0.

Напротив, высказывание «Саратов находится на берегу Невы, если и только если А.С.Пушкин — великий русский математик» истинно, так как оно является эквивалентностью двух ложных высказываний.


Союзы языка и логические операции (язык и логика)

Итак, каждая из введенных логических операций является неким математическим образом, моделью соответствующего логического союза нашего языка. Эти понятия призваны отразить на языке нулей и единиц соответствующие союзы нашего мышления, которыми человечество пользуется в течение тысячелетий. Вне всякого сомнения, язык нулей и единиц значительно беднее человеческого языка, и это отражение достаточно грубо и несовершенно. Тем не менее какие-то основные черты (существенные аспекты процессов мышления) понятия логических операций все же отражают. Так, отрицание, конъюнкция и эквивалентность достаточно точно передают суть логических союзов «не«, «и«, «тогда и только тогда, когда» соответственно. Хуже обстоит дело с дизъюнкцией, призванной отразить языковый союз «или«. Следует отметить, что кроме рассматриваемой так называемой дизъюнкции в не исключающем смысле (она истинна тогда и только тогда, когда по меньшей мере один ее член истинен) некоторые авторы рассматривают дизъюнкцию в исключающем смысле (или строгую дизъюнкцию): она истинна тогда и только тогда, когда истинен точно один ее член.

Наименее адекватным соответствующему союзу языка является понятие импликации, которое призвано отразить логический союз «если…, то…«. Это и понятно: на этом союзе основан один из сложнейших умственных процессов — процесс построения выводов, умозаключений. Импликация остается все же самой «коварной» из всех логических операций, и ее определение при всех приведенных доводах оставляет в нас чувство незавершенности. И это неспроста. Наиболее наглядно эта неадекватность определения языку проявится в ходе развития алгебры высказываний, когда мы, например, придем к тому, что тавтологией окажется следующая формула: (Pto Q)lor (Qto P). Это означает, что какие бы ни были высказывания P и Q, по меньшей мере одно из высказываний Pto Q или Qto P непременно будет истинным. Этот факт уже не согласуется с общепринятой практикой, и он еще раз подтверждает, что понятие импликации лишь весьма условно и приблизительно переводит на язык нулей и единиц тот смысл, который имеется в виду при построении фразы типа «если…, то…«.

Из приведенного следует вывод о том, что тонкое и многообразное человеческое мышление не так легко поддается научному осмыслению и изучению и что алгебра высказываний — всего лишь одно из приближений, всего лишь шаг на пути к познанию человеческого мышления.

По поводу происхождениия терминов отметим, что «конъюнкция» происходит от лат. conjunctio — соединение, дизъюнкция — от лат. dusjunctio — разъединение, импликация от лат. implicatio — сплетение и itnplico — тесно связываю.


Общий взгляд на логические операции

Еще раз отметим, что только логические значения или значения истинности, а не их содержание интересуют нас в развиваемой теории. Поэтому каждое из введенных определений (1.1, 1.3, 1.5, 1.7, 1.9) операций над высказываниями можно рассматривать как определение некоторого действия над символами 0 и 1, т. е. как определение некоторой операции на двухэлементном множестве {0;1}. Например, отрицание задает следующие правила действия с этими символами: lnot0=1,~ lnot1=0, конъюнкция — следующие: 0land0=0, 0land1=0, 1land0=0 1land1=1, импликация — следующие:

0to0=1,~~ 0to1=1,~~ 1to0=0,~~ 1to1=1 и т.д.

Учитывая два правила действия с символами 0 и 1, определяемые отрицанием, можно записать равенство для вычисления логического значения высказывания lnot P:

lambda(lnot P)= lnot lambda(P).

(1.1)

Указанные четыре правила действия с символами 0 и 1, определяемые конъюнкцией, позволяют записать равенство для вычисления логического значения высказывания Pland Q:

lambda(Pland Q)= lambda(P)land lambda(Q).

(1.2)

Аналогично, правила действия с символами 0 и 1, сформулированные в определениях 1.5, 1.7, 1.9, дают возможность записать равенства для вычисления логических значений высказываний Plor Q, Pto Q и P leftrightarrow Q соответственно:

lambda(Plor Q)= lambda(P)lor lambda(Q);

(1.3)

lambda(Pto Q)= lambda(P)to lambda(Q);

(1.4)

lambda(Pleftrightarrow Q)= lambda(P) leftrightarrow lambda(Q).

(1.5)

Равенства (1.2) … (1.5) можно записать в виде одного соотношения: lambda(Past Q)= lambda(P)ast lambda(Q), где значок «ast» обозначает один из символов логических операций land,lor,to,leftrightarrow. Равенства (1.1)–(1.5) фактически использовались при вычислениях логических значений высказываний

lnot A_6,quad A_2land A_3,quad A_3lor A_5,quad A_2lor A_7,quad A_6to A_5,quad A_2to A_7,quad A_5leftrightarrow A_8.

которые были проделаны выше в качестве примеров применения операций над высказываниями.

Математический форум (помощь с решением задач, обсуждение вопросов по математике).

Кнопка "Поделиться"

Если заметили ошибку, опечатку или есть предложения, напишите в комментариях.

Понравилась статья? Поделить с друзьями:
  • Как составить претензию на возврат денег за мебель
  • Как найти айфон без ведома владельца
  • Как найти корень уравнения если дискриминант отрицательный
  • Как составить план деятельности сотрудника
  • Как найти в телефоне музыку для звонка